6/9/2017 Endocrine ­ FRCEM Success

Dashboard Subscription expires in: 1 Days Extend

You have scored 24%

You answered 34 correct out of 144 questions.

Your answers are shown below:

Effects of thyroid hormones include all but which one of the following:

a) Increased heart rate b) Increased stroke volume c) Increased lipolysis d) Decreased gluconeogenesis e) Increased glycolysis

Something wrong?

The release of thyroid hormones is regulated by the anterior pituitary gland which secretes thyroid-stimulating hormone (TSH) and the hypothalamus which secretes thyrotropin-releasing hormone (TRH).

The thyroid follicles secrete two hormones; thyroxine (T4) – a prohormone that acts as a plasma reservoir and tri- iodothyronine (T3) – the active hormone. T3 and T4 synthesis involves the processing of tyrosine and iodine.

About 90% of thyroid hormones are secreted in the form of T4, with the remainder as T3. About 80% of the T4 is converted to the more active T3 (under stimulation of TSH) in the liver and .

Secretion of the thyroid hormones is stimulated by long-term exposure to cold temperatures acting on the anterior pituitary, oestrogens acting on the anterior pituitary and adrenaline acting directly on the thyroid gland. Increased serum levels of T3 inhibit secretion of TSH.

Most of the thyroid hormones in the blood are bound to plasma proteins (of these, 70% are bound to thyroid-binding globulin (TBG) and 30% are bound to albumin), which allows them to circulate without being broken down by enzymes. http://frcemsuccess.com/rev/sc­26/ 1/110 6/9/2017 Endocrine ­ FRCEM Success

Only a fraction of the circulating thyroid hormones (about 0.1% of T4 and 1% of T3) are unbound and thus biologically active. The thyroid hormones act to increase the basal metabolic rate and are important for growth and normal foetal development. Effects of thyroid hormones include: increased heart rate and stroke volume, and increased lipolysis, glycolysis and gluconeogenesis.

Which of the following is NOT an endocrine cause of :

a) Pseudohypoparathyroidism b) Hyperthyroidism c) Pheochromocytoma d) Addison’s disease e) Acromegaly

Something wrong?

Causes of hypercalcaemia include:

primary hyperparathyroidism (most commonly due to a benign parathyroid adenoma) familial hypocalciuric hypercalcaemia sudden acidosis (resulting in release of bound calcium) malignancy sarcoidosis chronic renal failure thyrotoxicosis pheochromocytoma Addison’s disease acromegaly ingestion of excess calcium (milk-alkali syndrome) or vitamin D drugs e.g. lithium, thiazide diuretics, theophylline toxicity prolonged immobilisation bone destruction usually by secondary deposits from malignancy or myeloma hypermagnesaemia

Pseudohypoparathyroidism (PHP) is a heterogeneous group of disorders characterised by , , increased serum concentration of parathyroid hormone (PTH), and insensitivity to the biologic activity of PTH.

The commonest causes of hypercalcaemia are primary hyperparathyroidism (most common) and hypercalcaemia of malignancy.

Features of hypercalcaemia include:

STONES (renal calculi) BONES (bone pain, fragile bones) GROANS (lethargy, fatigue, generalised muscle weakness) http://frcemsuccess.com/rev/sc­26/ 2/110 6/9/2017 Endocrine ­ FRCEM Success GROANS (lethargy, fatigue, generalised muscle weakness) MOANS (constipation, abdominal pain, nausea/vomiting, pancreatitis, peptic ulcer) THRONES (polyuria, polydipsia, dehydration, renal failure) PSYCHIATRIC OVERTONES (confusion, depression, anxiety, hallucinations, insomnia)

Parathyroid hormone (PTH) release is primarily simulated by which of the following:

a) High serum Ca2+ b) Low serum Ca2+ 3- c) High serum PO4 3- d) Low serum PO4 e) Activated vitamin D

Something wrong?

Parathyroid hormone (PTH) is synthesised by the chief cells in the parathyroid gland. PTH is released in response to falling plasma ionised calcium levels and increasing blood phosphate levels (indirectly by its binding to ionised calcium and thereby effective reduction of blood calcium levels).

PTH release is inhibited by normal blood calcium levels and hypomagnesaemia.

PTH acts to:

increase calcium and phosphate resorption from bone (via indirect upregulation of osteoclast activity) increase calcium reabsorption in the distal tubule of the nephron (by activating Ca2+ entry channels in the apical membrane and the Ca2+ ATPase pump on the basolateral membrane) increase phosphate excretion by inhibiting reabsorption in the proximal tubule of the nephron inhibit renal bicarbonate reabsorption stimulating a metabolism acidosis which favours dissociation of calcium from plasma proteins stimulate 1-alpha-hydroxylase in the kidneys to produce activated vitamin D indirectly increase calcium and phosphate absorption in the small intestine (via activated vitamin D)

Overall PTH acts to increase plasma calcium levels and decrease plasma phosphate levels.

Regarding the position of the pituitary gland, which of the following statements is CORRECT:

a) The pituitary gland lies in a bony hollow of the ethmoid bone. b) The pituitary gland is covered by the brous diaphragma sellae of the dura mater. c) The optic chiasm lies directly posterior to the anterior pituitary. d) The anterior pituitary is connected to the median eminence of the hypothalamus by the pituitary stalk. e) The sphenoid sinuses lie lateral to the pituitary gland.

Something wrong?

http://frcemsuccess.com/rev/sc­26/ 3/110 6/9/2017 Endocrine ­ FRCEM Success

The pituitary gland lies in a bony hollow of the sphenoid bone (the sella turcica), and it is covered by the brous diaphragma sellae of the dura mater. The optic chiasm lies directly superior to the anterior pituitary. The posterior pituitary is connected to the median eminence of the hypothalamus by the pituitary stalk (also known as the infundibulum). The cavernous sinuses (including cranial nerves III – VI) lie lateral to the pituitary gland.

Insulin secretion is stimulated by all but which one of the following:

a) Fatty acids and ketones b) Glucagon c) Secretin d) Somatostatin e) Growth hormone

Something wrong?

Insulin is a polypeptide hormone consisting of two short chains (A and B) linked by disulphide bonds.

Insulin is formed from the cleaving of proinsulin (derived from preproinsulin synthesised in the rough endoplasmic reticulum) into insulin and C-peptide in the Golgi body of β cells in the islets of Langerhans.

Since insulin and C-peptide are produced in equimolar amounts, C-peptide acts as a useful marker of β cell activity in diabetics who receive insulin treatment.

Insulin secretion is stimulated directly by high blood glucose levels, but also by metabolites such as amino acids, fatty acids and ketones, by glucagon, some gastrointestinal tract peptides (e.g. secretin), GH, ACTH and TSH. Insulin secretion is inhibited by low blood glucose levels, adrenaline, somatostatin, hypocalcaemia and sympathetic innervation. (N.B. insulin secretion never ceases completely, there is always a basal level of insulin in the blood)

Which of the following is NOT an expected nding in hyperprolactinaemia:

a) Amenorrhoea b) Failure of postpartum lactation c) Infertility d) Loss of libido e) Erectile dysfunction

Something wrong?

Prolactin acts on the mammary glands and reproductive organs to promote growth of these organs and initiate lactation. http://frcemsuccess.com/rev/sc­26/ 4/110 6/9/2017 Endocrine ­ FRCEM Success lactation.

Prolactin secretion is stimulated by prolactin-releasing factor (PRF) and thyrotropin-releasing hormone (TRH) from the hypothalamus. Prolactin secretion is inhibited by dopamine secreted by the hypothalamus.

Prolactin levels rise physiologically in pregnancy, puerperium, and breast stimulation.

Excess levels of prolactin may be caused by:

a prolactinoma (prolactin-secreting pituitary adenoma) compression of the pituitary stalk by a pituitary or hypothalamic tumour preventing normal dopaminergic inhibition of prolactin release head injury neurosurgery hypothyroidism (due to increased synthesis of TRH) Cushing’s syndrome severe liver disease polycystic ovary syndrome (PCOS) drugs e.g. dopamine antagonists, antidepressants.

Hyperprolactinaemia causes symptoms such as oligomenorrhoea/amenorrhoea, galactorrhoea, loss of libido, erectile dysfunction and infertility (via inhibition of the release of GnRH from the hypothalamus).

Prolactin deciency results in failure of postpartum lactation.

Which of the following is NOT a well known cause of hypoglycaemia:

a) Addison’s disease b) Propranolol c) Hypothermia d) Cushing’s syndrome e) Alcohol

Something wrong?

Hypoglycaemia is dened as a level of glucose < 3 mmol/L, although generally patients are not symptomatic unless plasma glucose reaches < 2.2 mmol/L.

It can occur due to:

oral hypoglycaemic drugs excess exogenous insulin alcohol hepatic failure renal impairment http://frcemsuccess.com/rev/sc­26/ 5/110 6/9/2017 Endocrine ­ FRCEM Success overwhelming sepsis hypothermia Addison’s disease insulinoma malignancy certain medications e.g. beta blockers or salicylates inadequate levels of exercise or carbohydrate intake in patients with diabetes mellitus

Symptoms of hypoglycaemia result from a catecholamine surge and include: feelings of hunger, tremor, nausea, sweating, anxiety/irritability, pallor, tachycardia or palpitations, headaches, tingling in the extremities or lips.

Symptoms of cognitive impairment reect neuroglycopenia and include confusion, poor concentration, seizures and ultimately coma.

Patients should be given oral glucose if possible (e.g. Lucozade or Glucogel), or if the oral route is not possible, intravenous dextrose if there is IV access (e.g. 75-80 ml 20% glucose or 150-160 ml of 10% glucose), or intramuscular/subcutaneous glucagon (e.g. 1 mg glucagon) if there is not.

Regarding the endocrine pancreas, which of the following statements is CORRECT:

a) The predominant endocrine cell type is the beta-cell. b) Delta-cells secrete glucagon. c) Endocrine hormones are released from Langhans giant cells. d) Somatostatin stimulates the release of both insulin and glucagon. e) Somatostatin is released from pancreatic acinar cells.

Something wrong?

The endocrine cells of the pancreas are arranged in small clusters around the larger exocrine cell clusters, called acini.

The endocrine clusters are called islets of Langerhans and within them are four types of cells:

1. alpha cells (comprise 20% of all cells) secrete glucagon 2. beta cells (70%) secrete insulin 3. delta cells (8%) secrete somatostatin 4. F-cells ( 2%) secrete pancreatic polypeptide.

Insulin and glucagon regulate blood glucose levels, somatostatin inhibits the release of both insulin and glucagon, pancreatic polypeptide inhibits the exocrine functions of the pancreas.

Regarding insulin receptors, which of the following statements is CORRECT:

a) Insulin receptors are G-protein coupled receptors. b) Insulin receptors consist of two extracellular beta subunits and two transmembrane alpha subunits. http://frcemsuccess.com/rev/sc­26/ 6/110 6/9/2017 Endocrine ­ FRCEM Success b) Insulin receptors consist of two extracellular beta subunits and two transmembrane alpha subunits. c) Insulin is a steroid hormone which can readily cross the cell membrane. d) Insulin receptors are only present on cells of the liver. e) Insulin has its intracellular effects via activation of tyrosine kinase.

Something wrong?

Insulin receptors are present on most cells and they can be sequestered into the cell to inactivate them; they consist of two extracellular alpha (α) subunits which contain the insulin-binding site and two transmembrane beta (β) subunits. Insulin must act via cell surface receptors as it is a polypeptide hormone which cannot readily cross the cell membrane.

When insulin binds to the receptor, the beta subunit autophosphorylates and activates tyrosine kinase causing an intracellular cascade of phosphorylation to bring about its intracellular effects.

Insulin acts to:

stimulate peripheral tissue glucose uptake stimulate glycogenesis stimulate glycolysis stimulate amino acid uptake and protein synthesis inhibit glycogenolysis inhibit proteolysis stimulate lipogenesis inhibit lipolysis inhibit gluconeogenesis

Which of the following is NOT an endocrine cause of hyponatraemia:

a) Addison’s disease b) Conn’s syndrome c) Abrupt withdrawal of long-term corticosteroids d) Hypothyroidism e) SIADH

Something wrong?

Hyponatraemia is dened as a serum sodium concentration of below the reference interval of 133 – 146 mmol/L. A level < 120 mmol/L is considered severe.

Hyponatraemia can arise either because of a primary loss of sodium ions or more commonly, due to increased retention of water relative to sodium which effectively dilutes the concentration of sodium.

Patients with hyponatraemia can be divided into three categories on the basis of their ECF volume. http://frcemsuccess.com/rev/sc­26/ 7/110 6/9/2017 Endocrine ­ FRCEM Success

Hyponatraemia in hypovolaemic patients results from a true sodium decit (following loss of and water) and may be caused by:

Gastrointestinal loss e.g. diarrhoea, vomiting, stula Skin loss e.g. excessive sweating, dermatitis, burns Renal losses e.g. diuretic therapy, renal tubular disease, cerebral salt wasting, mineralocorticoid deciency (Addison’s disease) Inadequate oral intake (very rare)

Hyponatraemia in euvolaemic patients results from excessive water retention due to the inability to excrete a water load, which may be caused by:

Acute increase in ADH following trauma/surgery/postpartum SIADH psychogenic polydipsia chronic renal disease severe hypothyroidism glucocorticoid deciency e.g. anterior pituitary disease or abrupt withdrawal of long-term glucocorticoids

Hyponatraemia in oedematous patients is actually associated with a high total body sodium, but there is a greater volume of water retained relative to sodium. Causes include:

renal failure congestive cardiac failure malnutrition cirrhosis nephrotic syndrome

What is the most likely diagnosis in a patient who presents with polyuria, polydipsia, a raised plasma osmolality and a low urine osmolality:

a) Diabetes mellitus b) Addison’s disease c) Diabetes insipidus d) Conn’s syndrome e) SIADH

Something wrong?

Diabetes insipidus (DI) may result from a deciency of ADH secretion (cranial DI) or from an inappropriate renal response to ADH (nephrogenic DI).

As a result, uid reabsorption at the kidneys is impaired, resulting in large amounts of hypotonic, dilute urine being passed and subsequent polydipsia. http://frcemsuccess.com/rev/sc­26/ 8/110 6/9/2017 Endocrine ­ FRCEM Success passed and subsequent polydipsia.

It is associated with elevated plasma osmolality (> 300 mOsm/kg) and low urine osmolality (< 600 mOsm/kg).

An ADH stimulation test can distinguish between cranial and nephrogenic DI, as nephrogenic DI shows an inability to concentrate urine even after administration of ADH.

A man with hypocalcaemia is found to have hypoparathyroidism. Which of the following is NOT a cause of hypoparathyroidism:

a) Haemochromatosis b) DiGeorge syndrome c) Hyperphosphataemia d) Thyroidectomy e) Magnesium deciency

Something wrong?

Causes of hypocalcaemia include:

Hypoparathyroidism (e.g. DiGeorge syndrome, autoimmune, secondary to parathyroid or thyroid surgery, secondary to radiation or chemotoxic agents, inltrative disease of the parathyroid glands e.g. haemochromatosis or secondary to hypomagnesaemia). Hyperphosphataemia may cause secondary hyperparathyroidism, NOT hypoparathyroidism. Pseudohypoparathyroidism (characterised by target organ resistance to PTH) Acute pancreatitis Renal failure Vitamin D deciency Alkalosis or massive blood transfusion (due to increased intravascular ionised calcium binding) Rhabdomyolysis or tumour lysis syndrome (due to extravascular calcium deposit) Drugs e.g. bisphosphates, uoride poisoning, combination chemotherapy Sepsis (due to impaired secretion of calcitriol and PTH, target organ resistance to PTH and increased lactate binding calcium in serum)

These causes can be remembered using the mnemonic HARVARDS.

The most common causes are hypoparathyroidism (frequently following surgery), vitamin D deciency or abnormal metabolism, chronic kidney disease and hypomagnesaemia.

Patients with adjusted serum calcium > 1.9 are usually asymptomatic but many patients become symptomatic when levels fall lower than this.

Features of hypocalcaemia include:

carpopedal spasm in response to inating a blood pressure cuff to above systolic BP (Trousseau’s sign) http://frcemsuccess.com/rev/sc­26/ 9/110 6/9/2017 Endocrine ­ FRCEM Success twitching of the ipsilateral facial muscles in response to tapping on the facial nerve (Chvostek’s sign) painful and fragile bones numbness and paraesthesia in hands and feet muscle cramps and spasms myopathy cardiac arrhythmias cardiac failure hypotension prolonged QT interval seizures mood changes cataracts

Insulin is secreted by which of the following pancreatic cell types:

a) α cells b) β cells c) δ cells d) F-cells e) Acinar cells

Something wrong?

The endocrine cells of the pancreas are arranged in small clusters around the larger exocrine cell clusters, called acini.

The endocrine clusters are called islets of Langerhans and within them are four types of cells:

1. alpha cells (comprise 20% of all cells) secrete glucagon 2. beta cells (70%) secrete insulin 3. delta cells (8%) secrete somatostatin 4. F-cells ( 2%) secrete pancreatic polypeptide.

Insulin and glucagon regulate blood glucose levels, somatostatin inhibits the release of both insulin and glucagon, pancreatic polypeptide inhibits the exocrine functions of the pancreas.

What is the underlying cause of Cushing’s disease:

a) Adrenal adenoma b) Ectopic ACTH c) Primary hyperaldosteronism d) Pituitary adenoma e) Exogenous steroids

Something wrong? http://frcemsuccess.com/rev/sc­26/ 10/110 6/9/2017 Endocrine ­ FRCEM Success

Cushing’s disease refers to the specic condition of excess corticosteroids as a result of increased ACTH due to a pituitary adenoma; the negative feedback that normally prevents excess ACTH secretion is absent in the tumour. Cushing’s disease is associated with hyperpigmentation due to the melanocyte-stimulating action of ACTH.

Glucagon acts to cause all but which one of the following effects:

a) Stimulation of glycogenolysis b) Increased peripheral tissue glucose uptake c) Stimulation of lipolysis d) Stimulation of ketogenesis e) Stimulation of gluconeogenesis

Something wrong?

Glucagon is synthesised in the pancreatic alpha (α) cells in the islets of Langerhans.

Glucagon secretion is stimulated by low blood glucose, amino acids, adrenaline, some gastrointestinal peptides, sympathetic and parasympathetic innervation. Glucagon secretion is inhibited by high blood glucose, fatty acids and ketones, insulin and somatostatin.

Glucagon acts on G-protein coupled receptors.

Glucagon acts to:

stimulate glycogenolysis inhibit peripheral tissue glucose uptake inhibit glycolysis inhibit amino acid uptake and protein synthesis stimulate lipolysis stimulate gluconeogenesis stimulate ketogenesis

Osteomalacia/rickets are associated with all but which one of the following clinical features:

a) Hypocalcaemia b) Bow-legs c) Bone pain d) Bone fragility and increased risk of fracture e) Hyperphosphataemia

Something wrong? http://frcemsuccess.com/rev/sc­26/ 11/110 6/9/2017 Endocrine ­ FRCEM Success

Osteomalacia is a disorder of mineralisation of bone matrix. Rickets is a disorder of defective mineralisation of cartilage in the epiphyseal growth plates of children.

Defective mineralisation is mostly due to reduced calcium and phosphate levels in the extracellular uid due to vitamin D deciency.

Features of osteomalacia include:

bone pain bones appear thin on X-ray with localised radiolucencies increased susceptibility to fractures features of hypocalcaemia

Features of Rickets include:

‘knock-knees’ or ‘bow-legs’ caused by bending of the long bones chest deformities back deformities e.g. kyphosis protruding forehead growth retardation

What is the most common cause of hyperthyroidism:

a) Toxic multinodular goitre b) Toxic thyroid adenoma c) TSH-secreting pituitary adenoma d) Graves’ disease e) Hashimoto’s thyroiditis

Something wrong?

Graves’ disease is an autoimmune disease, in which autoantibodies against TSH receptors are produced. These antibodies bind to and stimulate these TSH receptors leading to an excess production of thyroid hormones.

Graves’ disease is the most common cause of hyperthyroidism.

Classically Grave’s disease is associated with hyperthyroidism, pretibial myxoedema, clubbing (thyroid acropachy) and eye changes.

Graves’ eye disease may include features of: exophthalmos, lid retraction, lid lag, diplopia, corneal ulcers.

http://frcemsuccess.com/rev/sc­26/ 12/110 6/9/2017 Endocrine ­ FRCEM Success

Regarding antidiuretic hormone (ADH) which of the following statements is CORRECT:

a) ADH acts to increase water reabsorption by upregulating Na+ reabsorption. b) ADH has a potent vasodilator action at high doses. c) ADH release is stimulated by a fall in plasma volume. d) ADH release is stimulated by alcohol. e) Excess ADH results in diabetes insipidus.

Something wrong?

Antidiuretic hormone is synthesised in the hypothalamus and stored in secretory granules in the posterior pituitary. The main action on the kidneys is to increase the permeability of the distal tubule and the collecting duct to reabsorb water and thus increase water retention and concentrate urine.

ADH binds to V2 receptors on renal principal cells and increases cAMP, causing the incorporation of water channels called aquaporins into the apical membrane. It also has a potent vasoconstriction action at high doses.

ADH is rapidly removed from plasma, falling by about 50% in about 10 minutes, mainly due to metabolism in the liver and kidneys.

ADH release is stimulated by raised plasma osmolality (detected by osmoreceptors in the hypothalamus which also stimulate thirst) and a fall in blood pressure/plasma volume (detected by cardiac and vascular baroreceptors). The osmoreceptor system is more sensitive than the baroreceptor system.

ADH release is inhibited by low plasma osmolality, alcohol, caffeine, glucocorticoids and atrial natriuretic peptide (ANP).

ADH deciency (or an inadequate response to ADH) results in diabetes insipidus. Excess levels of ADH results in syndrome of inappropriate ADH secretion (SIADH).

Regarding hyponatraemia, which of the following statements is INCORRECT:

a) Hyponatraemia of any cause is usually associated with a low plasma osmolality. b) If plasma osmolality is normal in a patient with hyponatraemia, then the possibility of pseudohyponatraemia should be considered. c) Pseudohyponatraemia may be caused by hypoalbuminaemia. d) Hyperglycaemia may cause a hyperosmolar hyponatraemia. e) Mannitol may cause a hyperosmolar hyponatraemia.

Something wrong?

http://frcemsuccess.com/rev/sc­26/ 13/110 6/9/2017 Endocrine ­ FRCEM Success Hyponatraemia of any cause, is usually associated with a low plasma osmolality.

If the plasma osmolality is normal, then the possibility of pseudohyponatraemia should be considered; this is an artifactual result due to a reduction in plasma water caused by marked hyperlipidaemia or hyperproteinaemia e.g. multiple myeloma.

If the plasma osmolality is high, then the possibility of hyperosmolar hyponatraemia should be considered; this may be due to hyperglycaemia or administration of mannitol amongst other causes and reects the shift of water out of cells into the extracellular uid in response to osmotic effects.

Diabetes insipidus results from a deciency of or an inappropriate response to which of the following hormones:

a) Aldosterone b) Cortisol c) Angiotensin II d) Renin e) Antidiuretic hormone

Something wrong?

Diabetes insipidus (DI) may result from a deciency of ADH secretion (cranial DI) or from an inappropriate renal response to ADH (nephrogenic DI).

As a result, uid reabsorption at the kidneys is impaired, resulting in large amounts of hypotonic, dilute urine being passed and subsequent polydipsia.

It is associated with elevated plasma osmolality (> 300 mOsm/kg) and low urine osmolality (< 600 mOsm/kg).

An ADH stimulation test can distinguish between cranial and nephrogenic DI, as nephrogenic DI shows an inability to concentrate urine even after administration of ADH.

In a patient with hyponatraemia, dehydration and high urinary Na+, which of the following is the most likely diagnosis:

a) Vomiting b) Diabetes insipidus c) Nephrotic syndrome d) Sweating e) Diuretics

Something wrong?

http://frcemsuccess.com/rev/sc­26/ 14/110 6/9/2017 Endocrine ­ FRCEM Success

Hyponatraemia is dened as a serum sodium concentration of below the reference interval of 133 – 146 mmol/L. A Na + concentration < 120 mmol/L is considered severe.

Hyponatraemia can arise either because of a primary loss of sodium ions (in excess of water loss) or more commonly, due to increased retention of water relative to sodium which effectively dilutes the concentration of sodium (both are associated with a decrease in plasma osmolality).

Patients with hyponatraemia can be divided into three categories on the basis of their ECF volume.

Hyponatraemia in hypovolaemic patients results from a true sodium decit (following loss of salt > water) and may be caused by:

Prerenal Losses (urinary Na+ < 20 mmol/L) Gastrointestinal loss e.g. diarrhoea, vomiting, stula Skin loss e.g. excessive sweating, dermatitis, burns Third space loss e.g. burns, pancreatitis, cirrhosis Renal losses (urinary Na+ > 20 mmol/L) diuretic therapy osmotic diuresis renal tubular disease mineralocorticoid deciency in Addison’s disease Poor dietary salt intake (very rare)

Hyponatraemia in euvolaemic patients results from excessive water retention either due to:

increased intake of pure water (where urine osmolality is low and urinary Na+ < 20 mmol/L) for example: psychogenic polydipsia inappropriate intravenous dextrose administration inability to excrete a water load (where urine osmolality is high and urinary Na+ > 20 mmol/L) which may be caused by: SIADH chronic renal disease severe hypothyroidism glucocorticoid deciency e.g. anterior pituitary disease or abrupt withdrawal of long-term glucocorticoids

Hyponatraemia in oedematous patients is usually associated with a high total body sodium, but there is a greater volume of water retained relative to sodium. Causes include:

Where urinary Na+ > 20 mmol/L renal failure cerebral salt wasting inappropriate intravenous hypertonic saline Where urinary Na+ < 20 mmol/L congestive cardiac failure malnutrition cirrhosis http://frcemsuccess.com/rev/sc­26/ 15/110 6/9/2017 Endocrine ­ FRCEM Success cirrhosis nephrotic syndrome

Which of the following is NOT a direct effect of parathyroid hormone (PTH):

a) Increases Ca2+ reabsorption in the kidneys 3- b) Decreases PO4 reabsorption in the kidneys c) Increases bone resorption d) Stimulates activation of vitamin D by the kidneys e) Increases calcium reabsorption from the small intestine

Something wrong?

Parathyroid hormone (PTH) is synthesised by the chief cells in the parathyroid gland. PTH is released in response to falling plasma ionised calcium levels and increasing blood phosphate levels (indirectly by its binding to ionised calcium and thereby effective reduction of blood calcium levels).

PTH release is inhibited by normal blood calcium levels and hypomagnesaemia.

PTH acts to:

increase calcium and phosphate resorption from bone (via indirect upregulation of osteoclast activity) increase calcium reabsorption in the distal tubule of the nephron (by activating Ca2+ entry channels in the apical membrane and the Ca2+ ATPase pump on the basolateral membrane) increase phosphate excretion by inhibiting reabsorption in the proximal tubule of the nephron inhibit renal bicarbonate reabsorption stimulating a metabolism acidosis which favours dissociation of calcium from plasma proteins stimulate 1-alpha-hydroxylase in the kidneys to produce activated vitamin D indirectly increase calcium and phosphate absorption in the small intestine (via activated vitamin D)

Overall PTH acts to increase plasma calcium levels and decrease plasma phosphate levels.

How does parathyroid hormone (PTH) affect the renal handling of Ca2+:

a) Decreases calcium reabsorption in the proximal tubule b) Increases calcium ltration at the glomerulus c) Increases calcium reabsorption in the distal tubule d) Increases calcium reabsorption in the proximal tubule e) Decreases calcium reabsorption in the distal tubule

Something wrong?

http://frcemPsuacrcaetshsy.croomid/r ehvo/srcm­2o6n/ e (PTH) is synthesised by the chief cells in the parathyroid gland. PTH is released in response to 16/110 6/9/2017 Endocrine ­ FRCEM Success Parathyroid hormone (PTH) is synthesised by the chief cells in the parathyroid gland. PTH is released in response to falling plasma ionised calcium levels and increasing blood phosphate levels (indirectly by its binding to ionised calcium and thereby effective reduction of blood calcium levels).

PTH release is inhibited by normal blood calcium levels and hypomagnesaemia.

PTH acts to:

increase calcium and phosphate resorption from bone (via indirect upregulation of osteoclast activity) increase calcium reabsorption in the distal tubule of the nephron (by activating Ca2+ entry channels in the apical membrane and the Ca2+ ATPase pump on the basolateral membrane) increase phosphate excretion by inhibiting reabsorption in the proximal tubule of the nephron inhibit renal bicarbonate reabsorption stimulating a metabolism acidosis which favours dissociation of calcium from plasma proteins stimulate 1-alpha-hydroxylase in the kidneys to produce activated vitamin D indirectly increase calcium and phosphate absorption in the small intestine (via activated vitamin D)

Overall PTH acts to increase plasma calcium levels and decrease plasma phosphate levels.

Which of the following clinical features is NOT a classical feature of acromegaly:

a) Prognathism b) Spade-like hands c) Osteoarthritis d) Glucose intolerance e) Hypocalcaemia

Something wrong?

Acromegaly is a result of excess growth hormone (GH) secretion from the anterior pituitary gland, most commonly as a result of a GH-secreting pituitary adenoma.

Features of acromegaly include:

enlarged head circumference coarse and thickened skin resulting in prominent nasolabial folds, frontal bossing and supraorbital ridges large lower jaw (prognathism) spaces between lower teeth large nose macroglossia enlarged liver and kidneys spade-like hands mental disturbances insomnia loss of peripheral vision due to pituitary tumour compressing the optic chiasm (bitemporal hemianopia) http://frcemsuccess.com/rev/sc­26/ 17/110 6/9/2017 Endocrine ­ FRCEM Success loss of peripheral vision due to pituitary tumour compressing the optic chiasm (bitemporal hemianopia) cardiomyopathy hypertension osteoarthritis increased greasy sweating temperature intolerance hypercalcaemia glucose intolerance/diabetes mellitus predisposition to ischaemic heart disease carpal tunnel syndrome

In a euvolaemic patient with hyponatraemia and a urinary Na+ > 40 mmol/L, which of the following is the most likely diagnosis:

a) Nephrogenic diabetes insipidus b) Cranial diabetes insipidus c) Diabetes mellitus d) Syndrome of inappropriate ADH secretion e) Renal failure

Something wrong?

Hyponatraemia is dened as a serum sodium concentration of below the reference interval of 133 – 146 mmol/L. A Na + concentration < 120 mmol/L is considered severe.

Hyponatraemia can arise either because of a primary loss of sodium ions (in excess of water loss) or more commonly, due to increased retention of water relative to sodium which effectively dilutes the concentration of sodium (both are associated with a decrease in plasma osmolality).

Patients with hyponatraemia can be divided into three categories on the basis of their ECF volume.

Hyponatraemia in hypovolaemic patients results from a true sodium decit (following loss of salt > water) and may be caused by:

Prerenal Losses (urinary Na+ < 20 mmol/L) Gastrointestinal loss e.g. diarrhoea, vomiting, stula Skin loss e.g. excessive sweating, dermatitis, burns Third space loss e.g. burns, pancreatitis, cirrhosis Renal losses (urinary Na+ > 20 mmol/L) diuretic therapy osmotic diuresis renal tubular disease mineralocorticoid deciency in Addison’s disease Poor dietary salt intake (very rare) http://frcemsuccess.com/rev/sc­26/ 18/110 6/9/2017 Endocrine ­ FRCEM Success

Hyponatraemia in euvolaemic patients results from excessive water retention either due to:

increased intake of pure water (where urine osmolality is low and urinary Na+ < 20 mmol/L) for example: psychogenic polydipsia inappropriate intravenous dextrose administration inability to excrete a water load (where urine osmolality is high and urinary Na+ > 20 mmol/L) which may be caused by: SIADH chronic renal disease severe hypothyroidism glucocorticoid deciency e.g. anterior pituitary disease or abrupt withdrawal of long-term glucocorticoids

Hyponatraemia in oedematous patients is usually associated with a high total body sodium, but there is a greater volume of water retained relative to sodium. Causes include:

Where urinary Na+ > 20 mmol/L renal failure cerebral salt wasting inappropriate intravenous hypertonic saline Where urinary Na+ < 20 mmol/L congestive cardiac failure malnutrition cirrhosis nephrotic syndrome

Which of the following signs would you NOT expect to see in hypothyroidism:

a) Proximal myopathy b) Hyporeexia c) Positive Phalen’s test d) Bradycardia e) Pretibial myxoedema

Something wrong?

Hypothyroidism is dened as an underactive thyroid gland leading to decient thyroid hormones. T3 and T4 levels are low and TSH levels are usually high.

Causes include:

overtreatment of hyperthyroidism (e.g. radioactive ablation, surgical removal or anti-thyroid drugs e.g. carbimazole) drugs e.g. amiodarone, lithium, anticonvulsants, levodopa Hashimoto’s thyroiditis http://frcemsuccess.com/rev/sc­26/ 19/110 6/9/2017 Endocrine ­ FRCEM Success

De Quervain (subacute) thyroiditis primary atrophic hypothyroidism severe iodine deciency TSH deciency from hypopituitarism disruption of the hypothalamic-pituitary-thyroid axis

Features of hypothyroidism include:

coarse and thin hair loss of outer third of eyebrows mental slowing apathy lethargy psychosis myxoedemic features (i.e. pale puffy face, coarse features) hoarse voice swollen tongue deafness goitre bradycardia proximal myopathy constipation cold hands cold intolerance carpal tunnel syndrome slow reexes weight gain decreased sweating chronic oedema cold dry skin

TSH levels are normally elevated, low TSH levels suggest secondary hypothyroidism which is much less common.

Acromegaly results from an excess of which of the following hormones:

a) Cortisol b) Aldosterone c) Growth hormone d) ACTH e) Prolactin

Something wrong?

Acromegaly is a result of excess growth hormone (GH) secretion from the anterior pituitary gland, most commonly as a http://frcemrseucscuelst so.cf oam G/rHev-/sec­c2r6e/ting pituitary adenoma. 20/110 6/9/2017 Endocrine ­ FRCEM Success result of a GH-secreting pituitary adenoma.

Features of acromegaly include:

enlarged head circumference coarse and thickened skin resulting in prominent nasolabial folds, frontal bossing and supraorbital ridges large lower jaw (prognathism) spaces between lower teeth large nose macroglossia enlarged liver and kidneys spade-like hands mental disturbances insomnia loss of peripheral vision due to pituitary tumour compressing the optic chiasm (bitemporal hemianopia) cardiomyopathy hypertension osteoarthritis increased greasy sweating temperature intolerance hypercalcaemia glucose intolerance/diabetes mellitus predisposition to ischaemic heart disease carpal tunnel syndrome

Which of the following is NOT a well recognised feature of a pituitary adenoma:

a) Hypopituitarism b) Hyperpituitarism c) Homonymous hemianopia d) Ophthalmoplegia e) Hyperprolactinaemia

Something wrong?

Functioning pituitary adenomas cause disease by excess hormone release (hyperpituitarism).

Functioning pituitary adenomas usually release prolactin (most common type – about 50% of pituitary tumours), GH (about 20%) or ACTH (about 5%). TSH, LH and FSH – secreting adenomas are rare.

In addition to endocrine abnormalities, pituitary tumours can present with effects of a space-occupying lesion e.g. headaches, vomiting and papilloedema due to raised intracranial pressure, hypopituitarism from compression of normal secretory cells or compression of the portal veins that bring the hypothalamic-releasing factors (secretion of anterior pituitary hormones is inhibited in a characteristic order: GH, LH, FSH, ACTH, TSH, Prolactin (N.B. unless compression is severe, prolactin secretion is usually raised)) , visual eld defects (bitemporal hemianopia) from http://frcemsuccess.com/rev/sc­26/ 21/110 6/9/2017 Endocrine ­ FRCEM Success compression is severe, prolactin secretion is usually raised)) , visual eld defects (bitemporal hemianopia) from compression at the optic chiasm and cranial nerve palsies (CN III, IV, V and VI) from compression of the cavernous sinus.

ADH has which of the following actions:

a) Increases uid excretion by inhibiting reabsorption of sodium in the distal convoluted tubule b) Increases uid retention by increasing sodium reabsorption in the distal convoluted tubule c) Vasodilation d) Increases uid reabsorption in the proximal tubule via the insertion of aquaporin water channels e) Increases urine osmolality

Something wrong?

Antidiuretic hormone is synthesised in the hypothalamus and stored in secretory granules in the posterior pituitary. The main action on the kidneys is to increase the permeability of the distal tubule and the collecting duct to reabsorb water and thus increase water retention and concentrate urine.

ADH binds to V2 receptors on renal principal cells and increases cAMP, causing the incorporation of water channels called aquaporins into the apical membrane. It also has a potent vasoconstriction action at high doses.

ADH is rapidly removed from plasma, falling by about 50% in about 10 minutes, mainly due to metabolism in the liver and kidneys.

ADH release is stimulated by raised plasma osmolality (detected by osmoreceptors in the hypothalamus which also stimulate thirst) and a fall in blood pressure/plasma volume (detected by cardiac and vascular baroreceptors). The osmoreceptor system is more sensitive than the baroreceptor system.

ADH release is inhibited by low plasma osmolality, alcohol, caffeine, glucocorticoids and atrial natriuretic peptide (ANP).

ADH deciency (or an inadequate response to ADH) results in diabetes insipidus. Excess levels of ADH results in syndrome of inappropriate ADH secretion (SIADH).

What is the most likely diagnosis in a patient who presents with hypovolaemic hypotensive shock, who is found to have hyponatraemia and hyperkalaemia:

a) Adrenal crisis b) Thyrotoxic storm c) Pituitary apoplexy d) Myxoedema coma e) Pheochromocytoma

Something wrong? http://frcemsuccess.com/rev/sc­26/ 22/110 6/9/2017 Endocrine ­ FRCEM Success

Primary insufciency of the adrenal cortex is called Addison’s disease, it is characterised by decient secretion of both glucocorticoids and mineralocorticoids.

Causes include:

autoimmune adrenalitis (most common cause) infection e.g. tuberculosis (most common cause of Addison’s disease worldwide, but is rare in the UK) tumour e.g. metastases amyloidosis haemorrhagic necrosis of the adrenal gland (e.g. Waterhouse-Friderichsen syndrome secondary to DIC in meningococcal septicaemia)

High levels of circulating ACTH can cause hyperpigmentation.

Features of Addison’s disease include:

postural hypotension hyponatraemia hyperkalaemia hypoglycaemia tendency for hypercalcaemia weight loss abdominal pain/constipation/nausea muscle weakness fatigue lethargy dizziness depression

An acute exacerbation of Addison’s disease is called an adrenal crisis. It is a life-threatening emergency characterised by: hypotensive shock, hypovolaemic shock and hypoglycaemia.

Acute adrenocortical failure may occur if long-term high-dose steroid treatment is stopped abruptly (as the prolonged steroid treatment has suppressed natural ACTH release). Secondary adrenocortical insufciency may also be caused by disorders of the hypothalamus and anterior pituitary gland due to deciency of CRH or ACTH.

Glucagon secretion is stimulated by all but which one of the following:

a) Adrenaline b) Sympathetic innervation c) Parasympathetic innervation d) Fatty acids and ketones e) Low blood glucose http://frcemsuccess.com/rev/sc­26/ 23/110 6/9/2017 Endocrine ­ FRCEM Success

Something wrong?

Glucagon is synthesised in the pancreatic alpha (α) cells in the islets of Langerhans.

Glucagon secretion is stimulated by low blood glucose, amino acids, adrenaline, some gastrointestinal peptides, sympathetic and parasympathetic innervation. Glucagon secretion is inhibited by high blood glucose, fatty acids and ketones, insulin and somatostatin.

Glucagon acts on G-protein coupled receptors.

Glucagon acts to:

stimulate glycogenolysis inhibit peripheral tissue glucose uptake inhibit glycolysis inhibit amino acid uptake and protein synthesis stimulate lipolysis stimulate gluconeogenesis stimulate ketogenesis

Which of the following is NOT an effect stimulated by adrenaline:

a) Bronchodilation b) Increased heart rate c) Vasoconstriction of the coronary vessels d) Inhibited insulin release e) Increased renin release

Something wrong?

Adrenaline acts to:

cause alertness/agitation/fear/anxiety stimulate release of ACTH dilate pupils increase glycogenolysis increase renin release increase lipolysis increase sweating increase heart rate and contractility cause vasoconstriction in most tissues (but vasodilation in the coronary circulation) increase cardiac output and blood pressure http://frcemsuccedsisl.actoem t/rheev /bscr­o2n6c/ hioles 24/110 6/9/2017 Endocrine ­ FRCEM Success dilate the bronchioles stimulate ventilation stimulate glucagon release inhibit insulin release decrease proteolysis increase efciency of skeletal muscle contraction

Aldosterone is secreted by which of the following:

a) Zona glomerulosa of the adrenal cortex b) Zona reticularis of the adrenal cortex c) Zona fasciculata of the adrenal cortex d) Adrenal medulla e) Anterior pituitary

Something wrong?

The adrenal cortex is controlled by the pituitary gland, responding to adrenocorticotrophic (ACTH) hormone.

It is functionally and anatomically divided into three zones of tissue which each secrete different steroid hormones:

the outer zona glomerulosa which secretes mineralocorticoids (mainly aldosterone) the zona fasciculata which secretes glucocorticoids (mainly cortisol) the inner zona reticularis which secretes glucocorticoids and androgens (mainly dehydroepiandrosterone (DHEA))

The adrenal medulla produces catecholamines.

Which of the following clinical features would you NOT expect to see in hypothyroidism:

a) Loss of outer third of eyebrows b) Weight loss c) Pale puffy face d) Swollen tongue e) Goitre

Something wrong?

Hypothyroidism is dened as an underactive thyroid gland leading to decient thyroid hormones. T3 and T4 levels are low and TSH levels are usually high.

Causes include: http://frcemsuccess.com/rev/sc­26/ 25/110 6/9/2017 Endocrine ­ FRCEM Success

overtreatment of hyperthyroidism (e.g. radioactive ablation, surgical removal or anti-thyroid drugs e.g. carbimazole) drugs e.g. amiodarone, lithium, anticonvulsants, levodopa Hashimoto’s thyroiditis De Quervain (subacute) thyroiditis primary atrophic hypothyroidism severe iodine deciency TSH deciency from hypopituitarism disruption of the hypothalamic-pituitary-thyroid axis

Features of hypothyroidism include:

coarse and thin hair loss of outer third of eyebrows mental slowing apathy lethargy psychosis myxoedemic features (i.e. pale puffy face, coarse features) hoarse voice swollen tongue deafness goitre bradycardia proximal myopathy constipation cold hands cold intolerance carpal tunnel syndrome slow reexes weight gain decreased sweating chronic oedema cold dry skin

TSH levels are normally elevated, low TSH levels suggest secondary hypothyroidism which is much less common.

You are asked to assess a patient with hyponatraemia. They are hypervolaemic and have a low urinary sodium of < 20 mmol/L, which of the following is the most likely cause of their hyponatraemia:

a) SIADH b) Congestive cardiac heart failure c) Hypothyroidism d) Chronic kidney disease http://frcemsuccess.com/rev/sc­26/ 26/110 6/9/2017 Endocrine ­ FRCEM Success e) Psychogenic polydipsia

Something wrong?

Hyponatraemia is dened as a serum sodium concentration of below the reference interval of 133 – 146 mmol/L. A Na + concentration < 120 mmol/L is considered severe.

Hyponatraemia can arise either because of a primary loss of sodium ions (in excess of water loss) or more commonly, due to increased retention of water relative to sodium which effectively dilutes the concentration of sodium (both are associated with a decrease in plasma osmolality).

Patients with hyponatraemia can be divided into three categories on the basis of their ECF volume.

Hyponatraemia in hypovolaemic patients results from a true sodium decit (following loss of salt > water) and may be caused by:

Prerenal Losses (urinary Na+ < 20 mmol/L) Gastrointestinal loss e.g. diarrhoea, vomiting, stula Skin loss e.g. excessive sweating, dermatitis, burns Third space loss e.g. burns, pancreatitis, cirrhosis Renal losses (urinary Na+ > 20 mmol/L) diuretic therapy osmotic diuresis renal tubular disease mineralocorticoid deciency in Addison’s disease Poor dietary salt intake (very rare)

Hyponatraemia in euvolaemic patients results from excessive water retention either due to:

increased intake of pure water (where urine osmolality is low and urinary Na+ < 20 mmol/L) for example: psychogenic polydipsia inappropriate intravenous dextrose administration inability to excrete a water load (where urine osmolality is high and urinary Na+ > 20 mmol/L) which may be caused by: SIADH chronic renal disease severe hypothyroidism glucocorticoid deciency e.g. anterior pituitary disease or abrupt withdrawal of long-term glucocorticoids

Hyponatraemia in oedematous patients is usually associated with a high total body sodium, but there is a greater volume of water retained relative to sodium. Causes include:

Where urinary Na+ > 20 mmol/L renal failure cerebral salt wasting http://frcemsuccess.com/rev/sc­26/ 27/110 6/9/2017 Endocrine ­ FRCEM Success inappropriate intravenous hypertonic saline Where urinary Na+ < 20 mmol/L congestive cardiac failure malnutrition cirrhosis nephrotic syndrome

Where is vitamin D mainly activated in the body:

a) Skin b) Liver c) Thyroid gland d) Parathyroid gland e) Kidneys

Something wrong?

Vitamin D (cholecalciferol) is absorbed by the small intestine as part of the diet or is synthesised from cholesterol in skin.

Cholecalciferol is converted to calcifediol by 25-hydroxylase in the liver. This is converted to activated vitamin D (1, 25 – dihydroxycholecalciferol or calcitriol) by 1-alpha-hydroxylase in the kidney. The enzyme 1-alpha-hydroxylase is stimulated by parathyroid hormone (PTH) and by low concentrations of phosphate.

Activated vitamin D acts to:

increase calcium and phosphate absorption in the small intestine (the main action) increase renal calcium reabsorption (in the distal tubule via activation of a basolateral Ca2+ ATPase pump) increase renal phosphate reabsorption inhibit 1-alpha-hydroxylase activity in the kidneys (negative feedback) affect bone remodelling to bring about the conditions of high calcium and phosphate optimum for bone remineralisation

Vitamin D deciency can occur in:

dietary deciency malabsorption e.g. coeliac disease lack of sun exposure chronic kidney disease liver failure anticonvulsant therapy hypoparathyroidism

Deciency of vitamin D causes hypocalcaemia and osteomalacia (or rickets in children). http://frcemsuccess.com/rev/sc­26/ 28/110 6/9/2017 Endocrine ­ FRCEM Success

Regarding the gonadotropins, FSH and LH, which of the following statements is INCORRECT:

a) Gonadotrophin-releasing hormone is secreted from the hypothalamus. b) LH stimulates testicular production of testosterone. c) Gonadotropin release is stimulated by prolactin. d) Gonadotropins act via G-protein coupled receptors. e) Excess gonadotropins results in infertility.

Something wrong?

The gonadotropins, luteinising hormone (LH) and follicle stimulating hormone (FSH) act via G-protein coupled receptors on the gonads.

In the male, LH acts to stimulate production of testosterone, which acts in concert with FSH to support spermatogenesis. In the female, LH and FSH are essential for normal menstruation and reproduction.

LH/FSH secretion is stimulated by gonadotrophin-releasing hormone (GnRH) from the hypothalamus. GnRH is released in a pulsatile fashion, which is essential for normal reproductive activity.

LH/FSH secretion is inhibited by prolactin and sex steroids. LH/FSH deciency results in gonadal insufciency (decreased sex steroids). Excess levels of FSH/LH (extremely rare) results in infertility.

Which of the following is an action of oxytocin:

a) Stimulates Na+ and water reabsorption in the kidneys b) Simulates Ca2+ reabsorption in the small intestine c) Stimulates spermatogenesis d) Stimulates uterine contraction in childbirth e) Stimulates milk production in the mammary glands

Something wrong?

Oxytocin acts on the mammary glands to stimulate milk ejection, and the uterus to stimulate uterine contraction in childbirth. Oxytocin release is stimulated by stretch receptors in the nipple and the cervix and by oestrogen. Oxytocin release is inhibited by stress.

The adrenal medulla produces which of the following:

a) Mineralocorticoids b) Glucocorticoids c) Adrenal androgens http://frcemsuccess.com/rev/sc­26/ 29/110 6/9/2017 Endocrine ­ FRCEM Success c) Adrenal androgens d) Catecholamines e) Growth hormone

Something wrong?

The adrenal medulla produces catecholamines, and is controlled by and functions in concert with the sympathetic nervous system.

The chromafn cells of the medulla secrete noradrenaline (20%) and adrenaline (80%) stimulated by sympathetic preganglionic neurones located in the spinal cord in response to stress e.g. exercise, pain, shock, hypoglycaemia. These catecholamines act on alpha- and beta- G-protein coupled receptors , having the same effect in tissues as stimulation of sympathetic nerves.

Noradrenaline has equal potency at all adrenoceptors, but adrenaline at normal plasma concentrations will only activate beta-receptors (higher levels do stimulate alpha-receptors).

Noradrenaline is synthesised from the amino acid tyrosine which is then converted to adrenaline in response to cortisol from the adrenal cortex. Catecholamines circulate mainly bound to albumin and are broken down by monoamine oxidase (MAO) and catechol-O-methyl transferase (COMT) in the liver.

Regarding vitamin D, which of the following statements is CORRECT:

a) Vitamin D can only be obtained from the diet. b) Activation of vitamin D primarily occurs in the liver. c) Parathyroid hormone inhibits activation of vitamin D. d) Activated vitamin D increases renal excretion of phosphate. e) Vitamin D deciency can cause rickets in children.

Something wrong?

Vitamin D (cholecalciferol) is absorbed by the small intestine as part of the diet or is synthesised from cholesterol in skin.

Cholecalciferol is converted to calcifediol by 25-hydroxylase in the liver. This is converted to activated vitamin D (1, 25 – dihydroxycholecalciferol or calcitriol) by 1-alpha-hydroxylase in the kidney. The enzyme 1-alpha-hydroxylase is stimulated by parathyroid hormone (PTH) and by low concentrations of phosphate.

Activated vitamin D acts to:

increase calcium and phosphate absorption in the small intestine (the main action) increase renal calcium reabsorption (in the distal tubule via activation of a basolateral Ca2+ ATPase pump) increase renal phosphate reabsorption http://frcemsuccess.com/rev/sc­26/ 30/110 6/9/2017 Endocrine ­ FRCEM Success inhibit 1-alpha-hydroxylase activity in the kidneys (negative feedback) affect bone remodelling to bring about the conditions of high calcium and phosphate optimum for bone remineralisation

Vitamin D deciency can occur in:

dietary deciency malabsorption e.g. coeliac disease lack of sun exposure chronic kidney disease liver failure anticonvulsant therapy hypoparathyroidism

Deciency of vitamin D causes hypocalcaemia and osteomalacia (or rickets in children).

Regarding steroid hormones, which of the following statements is INCORRECT:

a) Steroid hormones are synthesised from cholesterol. b) Steroid hormones are small lipid-soluble molecules. c) Steroid hormones can readily cross cell membranes. d) Steroid hormones act on intracellular receptors to regulate gene expression. e) Steroid hormones travel mostly dissolved in plasma.

Something wrong?

Steroid hormones are synthesised from cholesterol. They are small lipid-soluble molecules that can readily cross cell membranes and that travel in plasma mainly bound to plasma protein. Inside cells, they act on intracellular receptors to regulate gene expression.

Calcitonin is primarily released in response to:

a) High serum Ca2+ b) Low serum Ca2+ 3- c) High serum PO4 3- d) Low serum PO4 e) Activated vitamin D

Something wrong?

Calcitonin is secreted by the parafollicular cells in the thyroid gland. Calcitonin is secreted in response to rising or high blood calcium levels and acts to lower circulating levels of calcium. http://frcemsuccess.com/rev/sc­26/ 31/110 6/9/2017 Endocrine ­ FRCEM Success blood calcium levels and acts to lower circulating levels of calcium.

Calcitonin acts on the kidneys to inhibit reabsorption of calcium and phosphate and on the bones to inhibit osteoclast activity and thus bone resorption of calcium and phosphate.

A patient’s blood results show hypernatraemia, with plasma osmolality > urine osmolality, which of the following is the most likely diagnosis:

a) SIADH b) Diabetes insipidus c) Conn’s syndrome d) Osmotic diuresis e) Inadequate water intake

Something wrong?

Hypernatraemia is an increase in the serum sodium concentration above the reference interval of 133 – 146 mmol/L. Hypernatraemia may arise from either excess sodium or much more commonly from water decit (both are associated with a raised plasma osmolality).

Hypernatraemia can be thought of in relation to actual total body sodium. The most common group of patients are those with hypernatraemia with decreased body sodium through loss of both water and sodium, but with a greater proportion of water loss which may result from:

renal losses e.g. osmotic diuresis in uncontrolled diabetes mellitus, loop diuretics, renal disease skin losses e.g. burns, excessive sweating in hot climate or exercise gastrointestinal losses e.g. vomiting, diarrhoea, stulae

Patients with hypernatraemia with normal total body sodium have a pure water decit which may result from:

inadequate water intake e.g. unconscious patient, dementia, disordered thirst perception in hypothalamic lesion excessive pure water loss e.g. diabetes insipidus (where plasma osmolality > urine osmolality)

Hypernatraemia with an actual increase in total body sodium is rare. Mild true hypernatraemia may be caused by primary hyperaldosteronism, but this is not typical of Conn’s syndrome, other causes include acute salt poisoning e.g. intravenous sodium bicarbonate, hypertonic saline, high sodium feeds in infants, near drowning in salt water.

When assessing patients with hypernatraemia:

If the hypernatraemia is mild (Na ≤ 150 mmol/L) and the patient has obvious signs of dehydration it is likely the ECF volume is reduced and that the patient has lost both sodium and water. Treatment should aim to replace the decit of uid by infusing isotonic saline, or if the decit is large, hypotonic saline. With more severe hypernatraemia (150 – 170 mmol/L), pure water loss is likely if the clinical signs of dehydration are mild in relation to the degree of hypernatraemia – this is because pure water loss is http://frcemsuccess.com/rev/sc­26/ 32/110 6/9/2017 Endocrine ­ FRCEM Success distributed evenly throughout the body compartments and the sodium content of the ECF is unchanged. Treatment should be aimed at replacing water either orally, or with 5% dextrose. With gross hypernatraemia ( > 180 mmol/L), an excess of sodium is likely, the patient may present with signs of uid overload. Treatment may be with diuretics, or rarely, by renal dialysis.

What is the most likely diagnosis in a young girl presenting with virilism and hirsutism:

a) Conn’s syndrome b) Cushing’s syndrome c) Congenital adrenal hyperplasia d) Addison’s disease e) Pheochromocytoma

Something wrong?

Congenital adrenal hyperplasia (CAH) is the result of an inherited enzyme defect in corticosteroid biosynthesis.

Because of the lack of cortisol, negative feedback to the pituitary is absent and ACTH secretion continues to drive steroid biosynthesis.

Cortisol precursors are secreted in large amounts. Increased stimulation of adrenal androgen production can cause virilisation in girls and precocious puberty in boys.

Regarding hyponatraemia, which of the following statements is CORRECT:

a) Hyponatraemia is dened as a serum Na+ concentration of less than 140 mmol/L. b) A level of Na+ < 130 mmol/L is considered severe hyponatraemia. c) Hyponatraemia most commonly occurs due to a true sodium decit. d) In patients with hyponatraemia, the plasma osmolality is usually high. e) Hyponatraemia in hypervolaemic patients is usually actually associated with a high total body Na+.

Something wrong?

Hyponatraemia is dened as a serum sodium concentration of below the reference interval of 133 – 146 mmol/L. A Na + concentration < 120 mmol/L is considered severe.

Hyponatraemia can arise either because of a primary loss of sodium ions (in excess of water loss) or more commonly, due to increased retention of water relative to sodium which effectively dilutes the concentration of sodium (both are associated with a decrease in plasma osmolality).

Patients with hyponatraemia can be divided into three categories on the basis of their ECF volume. http://frcemsuccess.com/rev/sc­26/ 33/110 6/9/2017 Endocrine ­ FRCEM Success

Hyponatraemia in hypovolaemic patients results from a true sodium decit (following loss of salt > water) and may be caused by:

Prerenal Losses (urinary Na+ < 20 mmol/L) Gastrointestinal loss e.g. diarrhoea, vomiting, stula Skin loss e.g. excessive sweating, dermatitis, burns Third space loss e.g. burns, pancreatitis, cirrhosis Renal losses (urinary Na+ > 20 mmol/L) diuretic therapy osmotic diuresis renal tubular disease mineralocorticoid deciency in Addison’s disease Poor dietary salt intake (very rare)

Hyponatraemia in euvolaemic patients results from excessive water retention either due to:

increased intake of pure water (where urine osmolality is low and urinary Na+ < 20 mmol/L) for example: psychogenic polydipsia inappropriate intravenous dextrose administration inability to excrete a water load (where urine osmolality is high and urinary Na+ > 20 mmol/L) which may be caused by: SIADH chronic renal disease severe hypothyroidism glucocorticoid deciency e.g. anterior pituitary disease or abrupt withdrawal of long-term glucocorticoids

Hyponatraemia in oedematous patients is usually associated with a high total body sodium, but there is a greater volume of water retained relative to sodium. Causes include:

Where urinary Na+ > 20 mmol/L renal failure cerebral salt wasting inappropriate intravenous hypertonic saline Where urinary Na+ < 20 mmol/L congestive cardiac failure malnutrition cirrhosis nephrotic syndrome

Which of the following symptoms would you NOT typically expect in hyperthyroidism:

a) Tachycardia b) Constipation c) Weight loss d) Heat intolerance e) Tremor http://frcemsuccess.com/rev/sc­26/ 34/110 6/9/2017 Endocrine ­ FRCEM Success e) Tremor

Something wrong?

Hyperthyroidism is dened as over activity of the thyroid gland, leading to excess thyroid hormone secretion.

Causes include:

Graves’ disease toxic multinodular goitre toxic adenoma ectopic thyroid tissue drugs e.g. amiodarone exogenous iodine

Features of hyperthyroidism include:

hair loss emotional lability fatigue anxiety restlessness goitre palpitations tachycardia atrial brillation proximal myopathy diarrhoea increased appetite weight loss tremor heat intolerance sweating oligomenorrhoea/amenorrhoea infertility reduced libido osteoporosis hyperreexia pretibial myxoedema (in Graves’ disease) palmar erythema clubbing onycholysis

T3 and T4 levels are elevated, and TSH levels are usually low. A raised TSH suggests the fault lies in or above the pituitary gland. http://frcemsuccess.com/rev/sc­26/ 35/110 6/9/2017 Endocrine ­ FRCEM Success

What is the pathophysiology of a pheochromocytoma:

a) GH-secreting tumour b) Catecholamine-secreting tumour c) Cortisol-secreting tumour d) ACTH-secreting tumour e) Aldosterone-secreting tumour

Something wrong?

A pheochromocytoma is a tumour usually of the adrenal medulla that leads to the excess production of catecholamines. They are usually benign and unilateral.

Features include (paroxysmal):

hypertension headaches profuse sweating palpitations tremor postural hypotension

What is the pathophysiology of Addison’s disease:

a) Adrenocortical insufciency b) Catecholamine-secreting tumour c) Chronic excess cortisol secretion d) ACTH-secreting tumour e) Aldosterone-secreting tumour

Something wrong?

Primary insufciency of the adrenal cortex is called Addison’s disease, it is characterised by decient secretion of both glucocorticoids and mineralocorticoids.

Causes include:

autoimmune adrenalitis (most common cause) infection e.g. tuberculosis (most common cause of Addison’s disease worldwide, but is rare in the UK) tumour e.g. metastases amyloidosis haemorrhagic necrosis of the adrenal gland (e.g. Waterhouse-Friderichsen syndrome secondary to DIC in http://frcemsuccess.com/rev/sc­26/ 36/110 6/9/2017 Endocrine ­ FRCEM Success haemorrhagic necrosis of the adrenal gland (e.g. Waterhouse-Friderichsen syndrome secondary to DIC in meningococcal septicaemia)

High levels of circulating ACTH can cause hyperpigmentation.

Features of Addison’s disease include:

postural hypotension hyponatraemia hyperkalaemia hypoglycaemia tendency for hypercalcaemia weight loss abdominal pain/constipation/nausea muscle weakness fatigue lethargy dizziness depression

An acute exacerbation of Addison’s disease is called an adrenal crisis. It is a life-threatening emergency characterised by: hypotensive shock, hypovolaemic shock and hypoglycaemia.

Acute adrenocortical failure may occur if long-term high-dose steroid treatment is stopped abruptly (as the prolonged steroid treatment has suppressed natural ACTH release). Secondary adrenocortical insufciency may also be caused by disorders of the hypothalamus and anterior pituitary gland due to deciency of CRH or ACTH.

Regarding catecholamines, which of the following statements is INCORRECT:

a) Catecholamines circulate mainly bound to albumin. b) Catecholamines are produced by the adrenal medulla. c) Catecholamines are broken down in the liver by COMT and MAO. d) Catecholamines act on alpha and beta G-protein coupled receptors. e) At normal plasma concentrations, adrenaline will only activate alpha receptors.

Something wrong?

The adrenal medulla produces catecholamines, and is controlled by and functions in concert with the sympathetic nervous system.

The chromafn cells of the medulla secrete noradrenaline (20%) and adrenaline (80%) stimulated by sympathetic preganglionic neurones located in the spinal cord in response to stress e.g. exercise, pain, shock, hypoglycaemia. These catecholamines act on alpha- and beta- G-protein coupled receptors , having the same effect in tissues as stimulation of sympathetic nerves.

http://frcemsuccess.com/rev/sc­26/ 37/110 6/9/2017 Endocrine ­ FRCEM Success Noradrenaline has equal potency at all adrenoceptors, but adrenaline at normal plasma concentrations will only activate beta-receptors (higher levels do stimulate alpha-receptors).

Noradrenaline is synthesised from the amino acid tyrosine which is then converted to adrenaline in response to cortisol from the adrenal cortex. Catecholamines circulate mainly bound to albumin and are broken down by monoamine oxidase (MAO) and catechol-O-methyl transferase (COMT) in the liver.

Regarding antidiuretic hormone (ADH) which of the following statements is CORRECT:

a) ADH is synthesised by the posterior pituitary gland. b) ADH primarily acts to increase water permeability in the proximal tubule. c) Excess ADH results in a reduction in urine osmolality. d) ADH binds to V2 receptors on renal principal cells. e) ADH has a half-life of about 6 – 8 hours.

Something wrong?

Antidiuretic hormone is synthesised in the hypothalamus and stored in secretory granules in the posterior pituitary. The main action on the kidneys is to increase the permeability of the distal tubule and the collecting duct to reabsorb water and thus increase water retention and concentrate urine.

ADH binds to V2 receptors on renal principal cells and increases cAMP, causing the incorporation of water channels called aquaporins into the apical membrane. It also has a potent vasoconstriction action at high doses.

ADH is rapidly removed from plasma, falling by about 50% in about 10 minutes, mainly due to metabolism in the liver and kidneys.

ADH release is stimulated by raised plasma osmolality (detected by osmoreceptors in the hypothalamus which also stimulate thirst) and a fall in blood pressure/plasma volume (detected by cardiac and vascular baroreceptors). The osmoreceptor system is more sensitive than the baroreceptor system.

ADH release is inhibited by low plasma osmolality, alcohol, caffeine, glucocorticoids and atrial natriuretic peptide (ANP).

ADH deciency (or an inadequate response to ADH) results in diabetes insipidus. Excess levels of ADH results in syndrome of inappropriate ADH secretion (SIADH).

Which of the following is NOT a typical feature of hypoglycaemia:

a) Tremor b) Palpitations c) Anxiety d) Polydipsia e) Perioral paraesthesia http://frcemsuccess.com/rev/sc­26/ 38/110 6/9/2017 Endocrine ­ FRCEM Success e) Perioral paraesthesia

Something wrong?

Hypoglycaemia is dened as a level of glucose < 3 mmol/L, although generally patients are not symptomatic unless plasma glucose reaches < 2.2 mmol/L.

It can occur due to:

oral hypoglycaemic drugs excess exogenous insulin alcohol hepatic failure renal impairment overwhelming sepsis hypothermia Addison’s disease insulinoma malignancy certain medications e.g. beta blockers or salicylates inadequate levels of exercise or carbohydrate intake in patients with diabetes mellitus

Symptoms of hypoglycaemia result from a catecholamine surge and include: feelings of hunger, tremor, nausea, sweating, anxiety/irritability, pallor, tachycardia or palpitations, headaches, tingling in the extremities or lips.

Symptoms of cognitive impairment reect neuroglycopenia and include confusion, poor concentration, seizures and ultimately coma.

Patients should be given oral glucose if possible (e.g. Lucozade or Glucogel), or if the oral route is not possible, intravenous dextrose if there is IV access (e.g. 75-80 ml 20% glucose or 150-160 ml of 10% glucose), or intramuscular/subcutaneous glucagon (e.g. 1 mg glucagon) if there is not.

What is the most likely diagnosis in a euvolaemic patient who has the following biochemical abnormalities; hyponatraemia, low plasma osmolality, high urine osmolality:

a) Diabetes insipidus b) Addison’s disease c) Conn’s syndrome d) SIADH e) Excess diuretic therapy

Something wrong?

Excess ADH (SIADH) causes water retention resulting in hypo-osmotic hyponatraemia. http://frcemsuccess.com/rev/sc­26/ 39/110 6/9/2017 Endocrine ­ FRCEM Success Excess ADH (SIADH) causes water retention resulting in hypo-osmotic hyponatraemia.

Typical biochemistry shows hyponatraemia, low plasma osmolality, high levels of urinary sodium and high urine osmolality.

Patients are euvolaemic and normotensive with normal thyroid and adrenal function. The potential causes are vast including:

Neurological: tumour, trauma, infection, Guillain-Barré syndrome, multiple sclerosis, systemic lupus erythematosus, intracranial haemorrhage, sinus thrombosis, AIDS, porphyria Pulmonary: lung small-cell cancer, mesothelioma, pneumonia, abscess, cystic brosis, asthma, tuberculosis, positive- pressure ventilation Other malignancy: oropharyngeal, stomach, pancreas, leukaemia, lymphoma, thymoma, and genitourinary tract cancers Drugs: chlorpropamide, carbamazepine, selective serotonin reuptake inhibitor (SSRI) antidepressants, tricyclic antidepressants, lithium, MDMA/ecstasy, tramadol, haloperidol, vincristine, desmopressin, uphenazine Miscellaneous: idiopathic, hereditary, pain, postoperative, stress, endurance exercise and marathon runners, dermatomal herpes zoster

Which of the following is NOT a well known cause of secondary diabetes mellitus:

a) Cushing’s syndrome b) Thyrotoxicosis c) Pheochromocytoma d) Acromegaly e) Diabetes insipidus

Something wrong?

“Fasting levels of glucose are normally 3.9 – 5.5 mmol/L, random levels are normally < 11 mmol/L.

Glucose homeostasis is primarily maintained by the interplay between insulin and glucagon. Insulin is the only hormone that can lower glucose levels but adrenaline, cortisol and growth hormone, can raise levels in addition to glucagon (by stimulating glycogenolysis and gluconeogenesis).

Hyperglycaemia is dened as a fasting level of glucose > 7 mmol/L. (Levels < 6 mmol/L are normal, levels 6 – 6.9 mmol/L demonstrate impaired fasting glycaemia, and levels >= 7 mmol/L are diagnostic of diabetes).

Hyperglycaemia may occur in diabetes mellitus type 1 due to a deciency of insulin (due to autoimmune destruction of β-cells), in diabetes mellitus type 2 due to a insufciency of insulin and/or insulin resistance or due to secondary causes of diabetes. http://frcemsuccess.com/rev/sc­26/ 40/110 6/9/2017 Endocrine ­ FRCEM Success of diabetes.

Secondary causes of diabetes include:

Cushing’s syndrome thyrotoxicosis pheochromocytoma acromegaly glucagonoma cirrhosis chronic pancreatitis cystic brosis pancreatectomy haemochromatosis pancreatic carcinoma thiazide diuretics corticosteroid use.

Clinical features of hyperglycaemia include:

polyuria glycosuria dehydration polydipsia tendency to infections lethargy weight loss wasting blurred vision weakness

You are asked to assess a patient with hyponatraemia. They are euvolaemic with a low plasma and urine osmolality and a urinary sodium of < 20 mmol/L, which of the following is the most likely cause of their hyponatraemia:

a) SIADH b) Diabetes insipidus c) Psychogenic polydipsia d) Hypothyroidism e) Renal failure

Something wrong?

http://frcemsuccess.com/rev/sc­26/ 41/110 6/9/2017 Endocrine ­ FRCEM Success Hyponatraemia is dened as a serum sodium concentration of below the reference interval of 133 – 146 mmol/L. A Na + concentration < 120 mmol/L is considered severe.

Hyponatraemia can arise either because of a primary loss of sodium ions (in excess of water loss) or more commonly, due to increased retention of water relative to sodium which effectively dilutes the concentration of sodium (both are associated with a decrease in plasma osmolality).

Patients with hyponatraemia can be divided into three categories on the basis of their ECF volume.

Hyponatraemia in hypovolaemic patients results from a true sodium decit (following loss of salt > water) and may be caused by:

Prerenal Losses (urinary Na+ < 20 mmol/L) Gastrointestinal loss e.g. diarrhoea, vomiting, stula Skin loss e.g. excessive sweating, dermatitis, burns Third space loss e.g. burns, pancreatitis, cirrhosis Renal losses (urinary Na+ > 20 mmol/L) diuretic therapy osmotic diuresis renal tubular disease mineralocorticoid deciency in Addison’s disease Poor dietary salt intake (very rare)

Hyponatraemia in euvolaemic patients results from excessive water retention either due to:

increased intake of pure water (where urine osmolality is low and urinary Na+ < 20 mmol/L) for example: psychogenic polydipsia inappropriate intravenous dextrose administration inability to excrete a water load (where urine osmolality is high and urinary Na+ > 20 mmol/L) which may be caused by: SIADH chronic renal disease severe hypothyroidism glucocorticoid deciency e.g. anterior pituitary disease or abrupt withdrawal of long-term glucocorticoids

Hyponatraemia in oedematous patients is usually associated with a high total body sodium, but there is a greater volume of water retained relative to sodium. Causes include:

Where urinary Na+ > 20 mmol/L renal failure cerebral salt wasting inappropriate intravenous hypertonic saline Where urinary Na+ < 20 mmol/L congestive cardiac failure malnutrition cirrhosis nephrotic syndrome http://frcemsuccess.com/rev/sc­26/ 42/110 6/9/2017 Endocrine ­ FRCEM Success nephrotic syndrome

What is the primary action of aldosterone:

a) Potassium reabsorption b) Sodium reabsorption c) Potassium secretion d) Sodium secretion e) Vasoconstriction

Something wrong?

Aldosterone is secreted by the zona glomerulosa of the adrenal cortex.

Aldosterone release is stimulated by:

angiotensin II a fall in extracellular uid volume (via the RAAS) a fall in plasma Na+ (via the RAAS) high plasma K+ ACTH

ACTH is less important as a regulator, so pituitary failure does not severely impair aldosterone secretion. Aldosterone acts mainly on the distal convoluted tubule (DCT) and the collecting duct of the kidney to cause increased reabsorption of sodium ions in exchange for potassium and hydrogen ions (via stimulation of Na+ pumps, Na+/H+ antiporters, and Na + and K+ channels in principal cells, and H+ ATPase in intercalated cells).

Water is also reabsorbed and blood volume therefore increased. Increased K+ excretion lowers plasma K+ levels. Aldosterone also stimulates Na+ conservation by the mucosal cells of the colon and gastric glands and by the sweat and salivary gland ducts.

You are asked to assess a patient with hyponatraemia. They are euvolaemic with a low plasma osmolality, a high urine osmolality and a urinary sodium of > 40 mmol/L, which of the following is the most likely cause of their hyponatraemia:

a) Psychogenic polydipsia b) SIADH c) Addison’s disease d) Diabetes insipidus e) Conn’s syndrome

Something wrong?

http://frcemsuccess.com/rev/sc­26/ 43/110 6/9/2017 Endocrine ­ FRCEM Success

Hyponatraemia is dened as a serum sodium concentration of below the reference interval of 133 – 146 mmol/L. A Na + concentration < 120 mmol/L is considered severe.

Hyponatraemia can arise either because of a primary loss of sodium ions (in excess of water loss) or more commonly, due to increased retention of water relative to sodium which effectively dilutes the concentration of sodium (both are associated with a decrease in plasma osmolality).

Patients with hyponatraemia can be divided into three categories on the basis of their ECF volume.

Hyponatraemia in hypovolaemic patients results from a true sodium decit (following loss of salt > water) and may be caused by:

Prerenal Losses (urinary Na+ < 20 mmol/L) Gastrointestinal loss e.g. diarrhoea, vomiting, stula Skin loss e.g. excessive sweating, dermatitis, burns Third space loss e.g. burns, pancreatitis, cirrhosis Renal losses (urinary Na+ > 20 mmol/L) diuretic therapy osmotic diuresis renal tubular disease mineralocorticoid deciency in Addison’s disease Poor dietary salt intake (very rare)

Hyponatraemia in euvolaemic patients results from excessive water retention either due to:

increased intake of pure water (where urine osmolality is low and urinary Na+ < 20 mmol/L) for example: psychogenic polydipsia inappropriate intravenous dextrose administration inability to excrete a water load (where urine osmolality is high and urinary Na+ > 20 mmol/L) which may be caused by: SIADH chronic renal disease severe hypothyroidism glucocorticoid deciency e.g. anterior pituitary disease or abrupt withdrawal of long-term glucocorticoids

Hyponatraemia in oedematous patients is usually associated with a high total body sodium, but there is a greater volume of water retained relative to sodium. Causes include:

Where urinary Na+ > 20 mmol/L renal failure cerebral salt wasting inappropriate intravenous hypertonic saline Where urinary Na+ < 20 mmol/L congestive cardiac failure malnutrition cirrhosis http://frcemsuccess.com/rev/sc­26/ 44/110 6/9/2017 Endocrine ­ FRCEM Success cirrhosis nephrotic syndrome

Adrenocorticotropic hormone (ACTH) primarily acts on which of the following target organs:

a) Anterior pituitary gland b) Posterior pituitary gland c) Adrenal medulla d) Adrenal cortex e) Liver

Something wrong?

Adrenocorticotrophic hormone (ACTH) acts on the adrenal cortex to stimulate glucocorticoid and androgen release.

ACTH secretion is stimulated by corticotrophin-releasing hormone (CRH) from the hypothalamus. ACTH secretion is inhibited by glucocorticoids.

Excess levels of ACTH due to a functioning pituitary adenoma results in Cushing’s disease.

ACTH deciency results in adrenocortical insufciency (decreased cortisol and adrenal androgens).

Regarding the adrenal medulla, which of the following statements is INCORRECT:

a) Catecholamines are produced by the chromafn cells of the adrenal medulla. b) The adrenal medulla predominantly secretes noradrenaline. c) The adrenal medulla is stimulated by acetylcholine released from preganglionic sympathetic neurones. d) Catecholamines are released by the adrenal medulla in response to stress. e) The released catecholamines act on G-protein coupled receptors.

Something wrong?

The adrenal medulla produces catecholamines, and is controlled by and functions in concert with the sympathetic nervous system.

The chromafn cells of the medulla secrete noradrenaline (20%) and adrenaline (80%) stimulated by sympathetic preganglionic neurones located in the spinal cord in response to stress e.g. exercise, pain, shock, hypoglycaemia. These catecholamines act on alpha- and beta- G-protein coupled receptors , having the same effect in tissues as stimulation of sympathetic nerves.

Noradrenaline has equal potency at all adrenoceptors, but adrenaline at normal plasma concentrations will only activate beta-receptors (higher levels do stimulate alpha-receptors). http://frcemsuccess.com/rev/sc­26/ 45/110 6/9/2017 Endocrine ­ FRCEM Success

Noradrenaline is synthesised from the amino acid tyrosine which is then converted to adrenaline in response to cortisol from the adrenal cortex. Catecholamines circulate mainly bound to albumin and are broken down by monoamine oxidase (MAO) and catechol-O-methyl transferase (COMT) in the liver.

Which of the following is NOT a risk factor for osteoporosis:

a) Hypoparathyroidism b) Thyroid disease c) Vitamin D deciency d) Post-menopausal women e) Family history of osteoporotic fractures

Something wrong?

Osteoporosis is a disease characterised by low bone mineral density and deterioration of bone sufcient to cause bone fragility and an increased risk of fracture (particularly hip, spine and distal radius).

It is caused by increased osteoclast activity and decreased osteoblast activity resulting in a shift towards increased bone resorption.

Risk factors include:

post-menopausal women family history of osteoporotic fractures rheumatoid arthritis thyroid disease long-term corticosteroid use high alcohol intake smoking vitamin D deciency hyperparathyroidism

Which of the following is NOT an effect of insulin:

a) Increased gluconeogenesis b) Decreased proteolysis c) Decreased glycogenolysis d) Increased lipogenesis e) Increased glycolysis

Something wrong?

http://frcemsuccess.com/rev/sc­26/ 46/110 6/9/2017 Endocrine ­ FRCEM Success

Insulin acts to:

stimulate peripheral tissue glucose uptake stimulate glycogenesis stimulate glycolysis stimulate amino acid uptake and protein synthesis inhibit glycogenolysis inhibit proteolysis stimulate lipogenesis inhibit lipolysis inhibit gluconeogenesis

Which of the following features is NOT typical of Addison’s disease:

a) Hyperpigmentation b) Hypotension c) Hypoglycaemia d) Hyperkalaemia e) Hypernatraemia

Something wrong?

Primary insufciency of the adrenal cortex is called Addison’s disease, it is characterised by decient secretion of both glucocorticoids and mineralocorticoids.

Causes include:

autoimmune adrenalitis (most common cause) infection e.g. tuberculosis (most common cause of Addison’s disease worldwide, but is rare in the UK) tumour e.g. metastases amyloidosis haemorrhagic necrosis of the adrenal gland (e.g. Waterhouse-Friderichsen syndrome secondary to DIC in meningococcal septicaemia)

High levels of circulating ACTH can cause hyperpigmentation.

Features of Addison’s disease include:

postural hypotension hyponatraemia hyperkalaemia hypoglycaemia tendency for hypercalcaemia weight loss http://frcemsuccess.com/rev/sc­26/ 47/110 6/9/2017 Endocrine ­ FRCEM Success abdominal pain/constipation/nausea muscle weakness fatigue lethargy dizziness depression

An acute exacerbation of Addison’s disease is called an adrenal crisis. It is a life-threatening emergency characterised by: hypotensive shock, hypovolaemic shock and hypoglycaemia.

Acute adrenocortical failure may occur if long-term high-dose steroid treatment is stopped abruptly (as the prolonged steroid treatment has suppressed natural ACTH release). Secondary adrenocortical insufciency may also be caused by disorders of the hypothalamus and anterior pituitary gland due to deciency of CRH or ACTH.

Glucagon is secreted by which of the following pancreatic cell types:

a) α cells b) β cells c) δ cells d) F cells e) Acinar cells

Something wrong?

The endocrine cells of the pancreas are arranged in small clusters around the larger exocrine cell clusters, called acini.

The endocrine clusters are called islets of Langerhans and within them are four types of cells:

1. α cells (comprise 20% of all cells) secrete glucagon 2. β cells (70%) secrete insulin 3. δ cells (8%) secrete somatostatin 4. F cells ( 2%) secrete pancreatic polypeptide.

Insulin and glucagon regulate blood glucose levels, somatostatin inhibits the release of both insulin and glucagon, pancreatic polypeptide inhibits the exocrine functions of the pancreas.

What is the most likely diagnosis in a 65 year old man presenting with hypertension, a large jaw and bitemporal hemianopia:

a) Cushing’s disease b) Acromegaly c) Graves’ disease d) Cranial diabetes insipidus e) Conn’s syndrome http://frcemsuccess.com/rev/sc­26/ 48/110 6/9/2017 Endocrine ­ FRCEM Success e) Conn’s syndrome

Something wrong?

Acromegaly is a result of excess growth hormone (GH) secretion from the anterior pituitary gland, most commonly as a result of a GH-secreting pituitary adenoma.

Features of acromegaly include:

enlarged head circumference coarse and thickened skin resulting in prominent nasolabial folds, frontal bossing and supraorbital ridges large lower jaw (prognathism) spaces between lower teeth large nose macroglossia enlarged liver and kidneys spade-like hands mental disturbances insomnia loss of peripheral vision due to pituitary tumour compressing the optic chiasm (bitemporal hemianopia) cardiomyopathy hypertension osteoarthritis increased greasy sweating temperature intolerance hypercalcaemia glucose intolerance/diabetes mellitus predisposition to ischaemic heart disease carpal tunnel syndrome

Blood glucose levels can be raised by all but which of the following hormones:

a) Cortisol b) Growth hormone c) Adrenaline d) Glucagon e) Insulin

Something wrong?

Fasting levels of glucose are normally 3.9 – 5.5 mmol/L, random levels are normally < 11 mmol/L.

Glucose homeostasis is primarily maintained by the interplay between insulin and glucagon. Insulin is the only http://frcemsuccess.com/rev/sc­26/ 49/110 6/9/2017 Endocrine ­ FRCEM Success hormone that can lower glucose levels but adrenaline, cortisol and growth hormone, can raise levels in addition to glucagon (by stimulating glycogenolysis and gluconeogenesis).

Hyperglycaemia is dened as a fasting level of glucose > 7 mmol/L. (Levels < 6 mmol/L are normal, levels 6 – 6.9 mmol/L demonstrate impaired fasting glycaemia, and levels >= 7 mmol/L are diagnostic of diabetes).

Hyperglycaemia may occur in diabetes mellitus type 1 due to a deciency of insulin (due to autoimmune destruction of β-cells), in diabetes mellitus type 2 due to a insufciency of insulin and/or insulin resistance or due to secondary causes of diabetes.

Secondary causes of diabetes include:

Cushing’s syndrome thyrotoxicosis pheochromocytoma acromegaly glucagonoma cirrhosis chronic pancreatitis cystic brosis pancreatectomy haemochromatosis pancreatic carcinoma thiazide diuretics corticosteroid use.

Clinical features of hyperglycaemia include:

polyuria glycosuria dehydration polydipsia tendency to infections lethargy weight loss wasting blurred vision weakness

What is the most likely diagnosis in a 35 year old woman who presents with hypertension, hypokalaemia and metabolic alkalosis:

a) Renal artery stenosis b) Cushing’s syndrome c) Cushing’s disease d) Conn’s syndrome http://frcemesu)ccess.com/rev/sc­26/ 50/110 6/9/2017 Endocrine ­ FRCEM Success e) Addison’s disease

Something wrong?

Hyperaldosteronism causes sodium and water retention (with volume expansion and hypertension) with increased excretion of potassium and hydrogen ions.

Features of hyperaldosteronism include:

hypertension hypokalaemia metabolic alkalosis polyuria and polydipsia muscle weakness and spasm N.B. Hypernatraemia rarely occurs due to other regulating mechanisms.

Conn’s syndrome is the most common cause of primary hyperaldosteronism, in which patients have an adenoma of the zona glomerulosa of the adrenal cortex.

In primary hyperaldosteronism, renin levels will be low due to negative feedback. High renin levels suggest secondary hyperaldosteronism e.g. secondary to excessive diuretic therapy (most common cause), congestive heart failure, renal artery stenosis, nephritic syndrome or cirrhosis with ascites.

Which of the following is NOT an effect stimulated by adrenaline:

a) Release of ACTH from the anterior pituitary b) Increased glycogenolysis in the liver c) Increased lipolysis d) Increased cardiac contractility e) Pupil constriction

Something wrong?

Adrenaline acts to:

cause alertness/agitation/fear/anxiety stimulate release of ACTH dilate pupils increase glycogenolysis increase renin release increase lipolysis increase sweating increase heart rate and contractility http://frcemsuccess.com/rev/sc­26/ 51/110 6/9/2017 Endocrine ­ FRCEM Success

cause vasoconstriction in most tissues (but vasodilation in the coronary circulation) increase cardiac output and blood pressure dilate the bronchioles stimulate ventilation stimulate glucagon release inhibit insulin release decrease proteolysis increase efciency of skeletal muscle contraction

What is the pathophysiology of Conn’s syndrome:

a) Adrenocortical insufciency b) Catecholamine-secreting tumour c) Chronic excess cortisol secretion d) ACTH-secreting tumour e) Aldosterone-secreting tumour

Something wrong?

Hyperaldosteronism causes sodium and water retention (with volume expansion and hypertension) with increased excretion of potassium and hydrogen ions.

Features of hyperaldosteronism include:

hypertension hypokalaemia metabolic alkalosis polyuria polydipsia muscle weakness spasm N.B. Hypernatraemia rarely occurs due to other regulating mechanisms

Conn’s syndrome is the most common cause of primary hyperaldosteronism, in which patients have an adenoma of the zona glomerulosa of the adrenal cortex. In primary hyperaldosteronism, renin levels will be low due to negative feedback. High renin levels suggest secondary hyperaldosteronism e.g. secondary to excessive diuretic therapy (most common cause), congestive heart failure, renal artery stenosis, nephritic syndrome or cirrhosis with ascites.

What is the most likely diagnosis in a 32 year old woman presenting with episodes of ushing, headaches, hypertension and palpitations:

a) Cushing’s syndrome b) Cushing’s disease c) Pheochromocytoma http://frcemsuccess.com/rev/sc­26/ 52/110 6/9/2017 Endocrine ­ FRCEM Success c) Pheochromocytoma d) Conn’s syndrome e) Hyperthyroidism

Something wrong?

A pheochromocytoma is a tumour usually of the adrenal medulla that leads to the excess production of catecholamines. They are usually benign and unilateral. Features include (paroxysmal): hypertension, headaches, profuse sweating, palpitations, tremor and postural hypotension.

Which of the following clinical features is NOT classical of Graves’ disease:

a) Exophthalmos b) Lid lag c) Ptosis d) Pretibial myxoedema e) Clubbing

Something wrong?

Graves’ disease is an autoimmune disease, in which autoantibodies against TSH receptors are produced. These antibodies bind to and stimulate these TSH receptors leading to an excess production of thyroid hormones. Graves’ disease is the most common cause of hyperthyroidism.

Classically Grave’s disease is associated with:

hyperthyroidism pretibial myxoedema clubbing (thyroid acropachy) eye changes including: exophthalmos, lid retraction, lid lag, diplopia, corneal ulcers

Which of the following hormones is secreted by the posterior pituitary gland:

a) Aldosterone b) Adrenocorticotropic hormone c) Angiotensin II d) Antidiuretic hormone e) Corticotrophin-releasing hormone

Something wrong?

http://frcemsuccess.com/rev/sc­26/ 53/110 6/9/2017 Endocrine ­ FRCEM Success

“The pituitary gland is primarily divided into two functional lobes, the anterior pituitary (adenohypophysis) and the posterior pituitary (neurohypophysis).

The following hormones are secreted from the anterior pituitary gland:

Follicle-stimulating hormone (FSH) Prolactin Thyroid-stimulating hormone (TSH) Adrenocorticotropic hormone (ACTH) Luteinising hormone (LH) Growth hormone (GH)

This can be remembered using the mnemonic: Fresh Pituitary Tastes Almost Like Guinness.

Antidiuretic hormone (ADH) and Oxytocin are secreted from the posterior pituitary gland.

The following pituitary hormones are matched with their main target tissue(s):

ACTH – Adrenal cortex GH – Liver, muscles, bones FSH – Gonads LH – Gonads Prolactin – Mammary glands TSH – Thyroid gland ADH – Kidney Oxytocin – Mammary glands, uterus

Which of the following features would you NOT expect in SIADH:

a) Hyponatraemia b) Low plasma osmolality c) High urinary sodium d) High urine osmolality e) Fluid overload

Something wrong?

Excess ADH (SIADH) causes water retention resulting in hypo-osmotic hyponatraemia.

Typical biochemistry shows hyponatraemia, low plasma osmolality, high levels of urinary sodium and high urine osmolality. http://frcemsuccess.com/rev/sc­26/ 54/110 6/9/2017 Endocrine ­ FRCEM Success Patients are euvolaemic and normotensive with normal thyroid and adrenal function. The potential causes are vast including:

Neurological: tumour, trauma, infection, Guillain-Barré syndrome, multiple sclerosis, systemic lupus erythematosus, intracranial haemorrhage, sinus thrombosis, AIDS, porphyria Pulmonary: lung small-cell cancer, mesothelioma, pneumonia, abscess, cystic brosis, asthma, tuberculosis, positive- pressure ventilation Other malignancy: oropharyngeal, stomach, pancreas, leukaemia, lymphoma, thymoma, and genitourinary tract cancers Drugs: chlorpropamide, carbamazepine, selective serotonin reuptake inhibitor (SSRI) antidepressants, tricyclic antidepressants, lithium, MDMA/ecstasy, tramadol, haloperidol, vincristine, desmopressin, uphenazine Miscellaneous: idiopathic, hereditary, pain, postoperative, stress, endurance exercise and marathon runners, dermatomal herpes zoster

You are asked to assess a patient with hyponatraemia. They are hypovolaemic with a low plasma osmolality, and a urinary sodium of < 20 mmol/L, which of the following is the most likely cause of their hyponatraemia:

a) Addison’s disease b) SIADH c) Diuretic therapy d) Gastroenteritis e) Poor oral intake due to dementia

Something wrong?

Hyponatraemia is dened as a serum sodium concentration of below the reference interval of 133 – 146 mmol/L. A Na + concentration < 120 mmol/L is considered severe.

Hyponatraemia can arise either because of a primary loss of sodium ions (in excess of water loss) or more commonly, due to increased retention of water relative to sodium which effectively dilutes the concentration of sodium (both are associated with a decrease in plasma osmolality).

Patients with hyponatraemia can be divided into three categories on the basis of their ECF volume.

Hyponatraemia in hypovolaemic patients results from a true sodium decit (following loss of salt > water) and may be caused by:

Prerenal Losses (urinary Na+ < 20 mmol/L) Gastrointestinal loss e.g. diarrhoea, vomiting, stula Skin loss e.g. excessive sweating, dermatitis, burns http://frcemsuccess.com/rev/sc­26/ 55/110 6/9/2017 Endocrine ­ FRCEM Success Skin loss e.g. excessive sweating, dermatitis, burns Third space loss e.g. burns, pancreatitis, cirrhosis Renal losses (urinary Na+ > 20 mmol/L) diuretic therapy osmotic diuresis renal tubular disease mineralocorticoid deciency in Addison’s disease Poor dietary salt intake (very rare)

Hyponatraemia in euvolaemic patients results from excessive water retention either due to:

increased intake of pure water (where urine osmolality is low and urinary Na+ < 20 mmol/L) for example: psychogenic polydipsia inappropriate intravenous dextrose administration inability to excrete a water load (where urine osmolality is high and urinary Na+ > 20 mmol/L) which may be caused by: SIADH chronic renal disease severe hypothyroidism glucocorticoid deciency e.g. anterior pituitary disease or abrupt withdrawal of long-term glucocorticoids

Hyponatraemia in oedematous patients is usually associated with a high total body sodium, but there is a greater volume of water retained relative to sodium. Causes include:

Where urinary Na+ > 20 mmol/L renal failure cerebral salt wasting inappropriate intravenous hypertonic saline Where urinary Na+ < 20 mmol/L congestive cardiac failure malnutrition cirrhosis nephrotic syndrome

Regarding insulin, which of the following statements is CORRECT:

a) Preproinsulin is synthesised in the smooth endoplasmic reticulum. b) Proinsulin is cleaved into insulin and protein B. c) Insulin consists of two short peptide chains linked by disulphide bonds. d) Insulin is secreted from alpha cells. e) Insulin secretion is directly stimulated by low blood glucose levels.

Something wrong?

Insulin is a polypeptide hormone consisting of two short chains (A and B) linked by disulphide bonds. http://frcemsuccess.com/rev/sc­26/ 56/110 6/9/2017 Endocrine ­ FRCEM Success Insulin is a polypeptide hormone consisting of two short chains (A and B) linked by disulphide bonds.

Insulin is formed from the cleaving of proinsulin (derived from preproinsulin synthesised in the rough endoplasmic reticulum) into insulin and C-peptide in the Golgi body of β cells in the islets of Langerhans.

Since insulin and C-peptide are produced in equimolar amounts, C-peptide acts as a useful marker of β cell activity in diabetics who receive insulin treatment.

Insulin secretion is stimulated directly by high blood glucose levels, but also by metabolites such as amino acids, fatty acids and ketones, by glucagon, some gastrointestinal tract peptides (e.g. secretin), GH, ACTH and TSH. Insulin secretion is inhibited by low blood glucose levels, adrenaline, somatostatin, hypocalcaemia and sympathetic innervation. (N.B. insulin secretion never ceases completely, there is always a basal level of insulin in the blood)

Which of the following acts to inhibit ADH release from the posterior pituitary:

a) Adrenaline b) Aldosterone c) Angiotensin II d) Adrenocorticotrophic hormone (ACTH) e) Atrial natriuretic peptide

Something wrong?

Antidiuretic hormone is synthesised in the hypothalamus and stored in secretory granules in the posterior pituitary. The main action on the kidneys is to increase the permeability of the distal tubule and the collecting duct to reabsorb water and thus increase water retention and concentrate urine.

ADH binds to V2 receptors on renal principal cells and increases cAMP, causing the incorporation of water channels called aquaporins into the apical membrane. It also has a potent vasoconstriction action at high doses.

ADH is rapidly removed from plasma, falling by about 50% in about 10 minutes, mainly due to metabolism in the liver and kidneys.

ADH release is stimulated by raised plasma osmolality (detected by osmoreceptors in the hypothalamus which also stimulate thirst) and a fall in blood pressure/plasma volume (detected by cardiac and vascular baroreceptors). The osmoreceptor system is more sensitive than the baroreceptor system.

ADH release is inhibited by low plasma osmolality, alcohol, caffeine, glucocorticoids and atrial natriuretic peptide (ANP).

ADH deciency (or an inadequate response to ADH) results in diabetes insipidus. Excess levels of ADH results in syndrome of inappropriate ADH secretion (SIADH).

Pituitary adenomas most commonly secrete which of the following hormones: http://frcemsuccess.com/rev/sc­26/ 57/110 6/9/2017 Endocrine ­ FRCEM Success

a) Growth hormone b) Adrenocorticotrophic hormone (ACTH) c) Thyroid-stimulating hormone (TSH) d) Prolactin e) Follicle-stimulating hormone

Something wrong?

Functioning pituitary adenomas cause disease by excess hormone release (hyperpituitarism).

Functioning pituitary adenomas usually release prolactin (most common type – about 50% of pituitary tumours), GH (about 20%) or ACTH (about 5%). TSH, LH and FSH – secreting adenomas are rare.

In addition to endocrine abnormalities, pituitary tumours can present with effects of a space-occupying lesion e.g. headaches, vomiting and papilloedema due to raised intracranial pressure, hypopituitarism from compression of normal secretory cells or compression of the portal veins that bring the hypothalamic-releasing factors (secretion of anterior pituitary hormones is inhibited in a characteristic order: GH, LH, FSH, ACTH, TSH, Prolactin (N.B. unless compression is severe, prolactin secretion is usually raised)) , visual eld defects (bitemporal hemianopia) from compression at the optic chiasm and cranial nerve palsies (CN III, IV, V and VI) from compression of the cavernous sinus.

What is the effect of activated vitamin D on the renal handling of calcium:

a) Decreases calcium reabsorption in the proximal tubule b) Increases calcium ltration at the glomerulus c) Increases calcium reabsorption in the distal tubule d) Increases calcium reabsorption in the proximal tubule e) Decreases calcium reabsorption in the distal tubule

Something wrong?

Vitamin D (cholecalciferol) is absorbed by the small intestine as part of the diet or is synthesised from cholesterol in skin.

Cholecalciferol is converted to calcifediol by 25-hydroxylase in the liver. This is converted to activated vitamin D (1, 25 – dihydroxycholecalciferol or calcitriol) by 1-alpha-hydroxylase in the kidney. The enzyme 1-alpha-hydroxylase is stimulated by parathyroid hormone (PTH) and by low concentrations of phosphate.

Activated vitamin D acts to:

increase calcium and phosphate absorption in the small intestine (the main action) increase renal calcium reabsorption (in the distal tubule via activation of a basolateral Ca2+ ATPase pump) http://frcemsuccess.com/rev/sc­26/ 58/110 6/9/2017 Endocrine ­ FRCEM Success increase renal calcium reabsorption (in the distal tubule via activation of a basolateral Ca ATPase pump) increase renal phosphate reabsorption inhibit 1-alpha-hydroxylase activity in the kidneys (negative feedback) affect bone remodelling to bring about the conditions of high calcium and phosphate optimum for bone remineralisation

Vitamin D deciency can occur in:

dietary deciency malabsorption e.g. coeliac disease lack of sun exposure chronic kidney disease liver failure anticonvulsant therapy hypoparathyroidism

Deciency of vitamin D causes hypocalcaemia and osteomalacia (or rickets in children).

Which of the following is NOT a secondary cause of hypertension:

a) Conn’s syndrome b) Pheochromocytoma c) SIADH d) Renal artery stenosis e) Acromegaly

Something wrong?

Causes of secondary hypertension include:

Cushing’s syndrome (excess glucocorticoids) Conn’s syndrome (excess aldosterone) Bilateral adrenal hyperplasia (excess aldosterone) Pheochromocytoma (excess catecholamines) Renal artery stenosis (excess renin causing secondary hyperaldosteronism) Acromegaly (excess growth hormone) Thyrotoxicosis (excess T3 and T4)

Which of the following stimulates thyroid-stimulating hormone (TSH) release from the anterior pituitary:

a) Raised serum levels of T3 b) Somatostatin c) Glucocorticoids d) Pregnancy http://frcemsuccess.com/rev/sc­26/ 59/110 6/9/2017 Endocrine ­ FRCEM Success d) Pregnancy e) Thyrotropin-releasing hormone (TRH)

Something wrong?

Thyroid-stimulating hormone (TSH) acts on the thyroid gland to stimulate thyroid hormone (T3 and T4) release.

TSH secretion is stimulated by thyrotropin-releasing hormone (TRH) from the hypothalamus.

TSH secretion is inhibited by raised serum levels of T3 or T4, somatostatin, dopamine, glucocorticoids, acute non- thyroidal illness and increased human chorionic gonadotrophin (e.g. in early pregnancy).

TSH deciency results in hypothyroidism. Excess TSH (extremely rare) results in hyperthyroidism.

Cortisol is secreted by which of the following:

a) Zona glomerulosa of the adrenal cortex b) Zona reticularis of the adrenal cortex c) Zona fasciculata of the adrenal cortex d) Adrenal medulla e) Anterior pituitary

Something wrong?

The adrenal cortex is controlled by the pituitary gland, responding to adrenocorticotrophic (ACTH) hormone.

It is functionally and anatomically divided into three zones of tissue which each secrete different steroid hormones:

the outer zona glomerulosa which secretes mineralocorticoids (mainly aldosterone) the zona fasciculata which secretes glucocorticoids (mainly cortisol) the inner zona reticularis which secretes glucocorticoids and androgens (mainly dehydroepiandrosterone (DHEA))

The adrenal medulla produces catecholamines.

What is the most likely diagnosis in a 46-year old woman presenting with buccal hyperpigmentation, postural hypotension and hyperkalaemia:

a) Pheochromocytoma b) Conn’s syndrome c) Cushing’s syndrome d) Congenital adrenal hyperplasia http://frcemesu)ccAedssd.cisoomn/’rse vd/sisce­2a6s/e 60/110 6/9/2017 Endocrine ­ FRCEM Success e) Addison’s disease

Something wrong?

Primary insufciency of the adrenal cortex is called Addison’s disease, it is characterised by decient secretion of both glucocorticoids and mineralocorticoids.

Causes include:

autoimmune adrenalitis (most common cause) infection e.g. tuberculosis (most common cause of Addison’s disease worldwide, but is rare in the UK) tumour e.g. metastases amyloidosis haemorrhagic necrosis of the adrenal gland (e.g. Waterhouse-Friderichsen syndrome secondary to DIC in meningococcal septicaemia)

High levels of circulating ACTH can cause hyperpigmentation.

Features of Addison’s disease include:

postural hypotension hyponatraemia hyperkalaemia hypoglycaemia tendency for hypercalcaemia weight loss abdominal pain/constipation/nausea muscle weakness fatigue lethargy dizziness depression

An acute exacerbation of Addison’s disease is called an adrenal crisis. It is a life-threatening emergency characterised by: hypotensive shock, hypovolaemic shock and hypoglycaemia.

Acute adrenocortical failure may occur if long-term high-dose steroid treatment is stopped abruptly (as the prolonged steroid treatment has suppressed natural ACTH release). Secondary adrenocortical insufciency may also be caused by disorders of the hypothalamus and anterior pituitary gland due to deciency of CRH or ACTH.

Which of the following is NOT recognised as a cause of hypothyroidism:

a) Sedating antihistamines b) Amiodarone c) Carbimazole http://frcemsuccess.com/rev/sc­26/ 61/110 6/9/2017 Endocrine ­ FRCEM Success c) Carbimazole d) Lithium e) Phenytoin

Something wrong?

Hypothyroidism is dened as an underactive thyroid gland leading to decient thyroid hormones. T3 and T4 levels are low and TSH levels are usually high.

Causes include:

overtreatment of hyperthyroidism (e.g. radioactive ablation, surgical removal or anti-thyroid drugs e.g. carbimazole) drugs e.g. amiodarone, lithium, anticonvulsants, levodopa Hashimoto’s thyroiditis De Quervain (subacute) thyroiditis primary atrophic hypothyroidism severe iodine deciency TSH deciency from hypopituitarism disruption of the hypothalamic-pituitary-thyroid axis

Features of hypothyroidism include:

coarse and thin hair loss of outer third of eyebrows mental slowing apathy lethargy psychosis myxoedemic features (i.e. pale puffy face, coarse features) hoarse voice swollen tongue deafness goitre bradycardia proximal myopathy constipation cold hands cold intolerance carpal tunnel syndrome slow reexes weight gain decreased sweating chronic oedema cold dry skin http://frcemsuccess.com/rev/sc­26/ 62/110 6/9/2017 Endocrine ­ FRCEM Success

TSH levels are normally elevated, low TSH levels suggest secondary hypothyroidism which is much less common.

Which of the following clinical features is NOT a classical feature of acromegaly:

a) Prominent nasolabial folds b) Macroglossia c) Bitemporal hemianopia d) Postural hypotension e) Carpal tunnel syndrome

Something wrong?

Acromegaly is a result of excess growth hormone (GH) secretion from the anterior pituitary gland, most commonly as a result of a GH-secreting pituitary adenoma.

Features of acromegaly include:

enlarged head circumference coarse and thickened skin resulting in prominent nasolabial folds, frontal bossing and supraorbital ridges large lower jaw (prognathism) spaces between lower teeth large nose macroglossia enlarged liver and kidneys spade-like hands mental disturbances insomnia loss of peripheral vision due to pituitary tumour compressing the optic chiasm (bitemporal hemianopia) cardiomyopathy hypertension osteoarthritis increased greasy sweating temperature intolerance hypercalcaemia glucose intolerance/diabetes mellitus predisposition to ischaemic heart disease carpal tunnel syndrome

What is the primary aim of calcitonin:

a) Increase plasma Ca2+ b) Decrease plasma Ca2+ 3- c) Increase plasma PO4 3- http://frcemsuccess.com/rev/sc­26/ 63/110 6/9/2017 Endocrine ­ FRCEM Success 3- d) Decrease plasma PO4 e) Increase plasma Mg2+

Something wrong?

Calcitonin is secreted by the parafollicular cells in the thyroid gland. Calcitonin is secreted in response to rising or high blood calcium levels and acts to lower circulating levels of calcium.

Calcitonin acts on the kidneys to inhibit reabsorption of calcium and phosphate and on the bones to inhibit osteoclast activity and thus bone resorption of calcium and phosphate.

Which of the following pituitary hormones is NOT paired correctly with its main target organ(s):

a) Oxytocin – Uterus b) Luteinising hormone – Gonads c) Thyroid-stimulating hormone – Thyroid d) Prolactin – Mammary glands e) Antidiuretic hormone – Adrenal cortex

Something wrong?

“The pituitary gland is primarily divided into two functional lobes, the anterior pituitary (adenohypophysis) and the posterior pituitary (neurohypophysis).

The following hormones are secreted from the anterior pituitary gland:

Follicle-stimulating hormone (FSH) Prolactin Thyroid-stimulating hormone (TSH) Adrenocorticotropic hormone (ACTH) Luteinising hormone (LH) Growth hormone (GH)

This can be remembered using the mnemonic: Fresh Pituitary Tastes Almost Like Guinness.

Antidiuretic hormone (ADH) and Oxytocin are secreted from the posterior pituitary gland.

The following pituitary hormones are matched with their main target tissue(s):

ACTH – Adrenal cortex GH – Liver, muscles, bones FSH – Gonads http://frcemsuccess.com/rev/sc­26/ 64/110 6/9/2017 Endocrine ­ FRCEM Success FSH – Gonads LH – Gonads Prolactin – Mammary glands TSH – Thyroid gland ADH – Kidney Oxytocin – Mammary glands, uterus

Which of the following would you NOT expect in diabetic ketoacidosis:

a) Acetone smell on the breath b) Ketonuria c) Hyperglycaemia d) Hypoventilation e) Raised anion gap metabolic acidosis

Something wrong?

Diabetic ketoacidosis is a life-threatening complication of diabetes mellitus.

In the absence of adequate amounts of insulin, the body is unable to use glucose for energy and so fatty acids are released from adipose tissue and converted to ketone bodies by the liver. The ketone bodies are acidic and result in a metabolic acidosis (and ketonaemia and ketonuria).

The excess glucose (due to decreased cellular glucose uptake) and the ketone bodies cause an osmotic diuresis resulting in dehydration and a loss of electrolytes (sodium, potassium, chloride, calcium, magnesium, phosphate).

Features of ketoacidosis include:

lethargy vomiting anorexia abdominal pain acetone smell on the breath ketonuria polydipsia polyuria dehydration hyperventilation (Kussmaul breathing) shock reduced consciousness coma death

http://frcemsuccess.com/rev/sc­26/ 65/110 6/9/2017 Endocrine ­ FRCEM Success The most common precipitating factors in the development of DKA are infection, myocardial infarction, trauma or omission of insulin.

The management of DKA requires the administration of insulin, uids and potassium (despite apparently normal serum potassium levels).

Which of the following is the most common cause of Cushing’s syndrome in adults:

a) ACTH-secreting pituitary adenoma b) Ectopic ACTH secreting small-cell carcinoma of the lung c) Adrenal adenoma d) Adrenal carcinoma e) Exogenous glucocorticoids

Something wrong?

Cushing’s syndrome is a rare condition caused by a chronic excess of glucocorticoids.

This may occur due to:

1. excess secretion of ACTH or CRH (e.g. ACTH secreting pituitary adenoma, ectopic ACTH secretion by small- cell carcinoma of the lung or bronchial carcinoid tumour) 2. excess secretion of cortisol (e.g. adenoma of the adrenal cortex (most common cause of Cushing’s syndrome in children) or adrenal carcinoma) 3. exogenous steroids (most common cause of Cushing’s syndrome in adults).

Features of Cushing’s syndrome include:

hair thinning male-pattern baldness cataracts truncal obesity striae dorsocervical fat pad (buffalo-hump) moon face acne hirsutism hypertension skeletal muscle weakness and wasting peptic ulceration renal calculi menstrual disturbance osteoporosis and associated increased risk of fracture/vertebral collapse skin thinning, easy bruising, tendency to skin infections http://frcemsucceasns.kcloem o/reedv/esmc­a26/ 66/110 6/9/2017 Endocrine ­ FRCEM Success ankle oedema glucose intolerance/DM predisposition to congestive cardiac failure depression/confusion/insomnia/psychosis

Which of the following hormones is released in direct response to high plasma [K+]:

a) Antidiuretic hormone b) Aldosterone c) Cortisol d) Adrenocorticotrophic hormone e) Renin

Something wrong?

Aldosterone is secreted by the zona glomerulosa of the adrenal cortex.

Aldosterone release is stimulated by:

angiotensin II a fall in extracellular uid volume (via the RAAS) a fall in plasma Na+ (via the RAAS) high plasma K+ ACTH

ACTH is less important as a regulator, so pituitary failure does not severely impair aldosterone secretion. Aldosterone acts mainly on the distal convoluted tubule (DCT) and the collecting duct of the kidney to cause increased reabsorption of sodium ions in exchange for potassium and hydrogen ions (via stimulation of Na+ pumps, Na+/H+ antiporters, and Na + and K+ channels in principal cells, and H+ ATPase in intercalated cells).

Water is also reabsorbed and blood volume therefore increased. Increased K+ excretion lowers plasma K+ levels. Aldosterone also stimulates Na+ conservation by the mucosal cells of the colon and gastric glands and by the sweat and salivary gland ducts.

Regarding osteomalacia, which of the following statements is CORRECT:

a) Osteomalacia is a disease characterised by loss of bone mineral density. b) Osteomalacia occurs most commonly as a result of primary hypoparathyroidism. c) Osteomalacia is associated with raised serum Ca2+. d) On X-ray bones appear dense with localised radiolucencies. e) Rickets is a disorder of defective mineralisation of cartilage in the epiphyseal growth plates of children.

Something wrong?

http://frcemsuccess.com/rev/sc­26/ 67/110 6/9/2017 Endocrine ­ FRCEM Success

Osteomalacia is a disorder of mineralisation of bone matrix. Rickets is a disorder of defective mineralisation of cartilage in the epiphyseal growth plates of children.

Defective mineralisation is mostly due to reduced calcium and phosphate levels in the extracellular uid due to vitamin D deciency.

Features of osteomalacia include:

bone pain bones appear thin on X-ray with localised radiolucencies increased susceptibility to fractures features of hypocalcaemia

Features of Rickets include:

‘knock-knees’ or ‘bow-legs’ caused by bending of the long bones chest deformities back deformities e.g. kyphosis protruding forehead growth retardation

ADH release from the posterior pituitary is directly stimulated by which of the following:

a) Rise in blood volume b) Rise in blood pressure c) Rise in plasma osmolality d) Rise in plasma Na+ concentration e) Rise in plasma K+ concentration

Something wrong?

Antidiuretic hormone is synthesised in the hypothalamus and stored in secretory granules in the posterior pituitary. The main action on the kidneys is to increase the permeability of the distal tubule and the collecting duct to reabsorb water and thus increase water retention and concentrate urine.

ADH binds to V2 receptors on renal principal cells and increases cAMP, causing the incorporation of water channels called aquaporins into the apical membrane. It also has a potent vasoconstriction action at high doses.

ADH is rapidly removed from plasma, falling by about 50% in about 10 minutes, mainly due to metabolism in the liver and kidneys.

ADH release is stimulated by raised plasma osmolality (detected by osmoreceptors in the hypothalamus which also stimulate thirst) and a fall in blood pressure/plasma volume (detected by cardiac and vascular baroreceptors). The http://frcemsuccess.com/rev/sc­26/ 68/110 6/9/2017 Endocrine ­ FRCEM Success stimulate thirst) and a fall in blood pressure/plasma volume (detected by cardiac and vascular baroreceptors). The osmoreceptor system is more sensitive than the baroreceptor system.

ADH release is inhibited by low plasma osmolality, alcohol, caffeine, glucocorticoids and atrial natriuretic peptide (ANP).

ADH deciency (or an inadequate response to ADH) results in diabetes insipidus. Excess levels of ADH results in syndrome of inappropriate ADH secretion (SIADH).

Which of the following is the most common cause of Addison’s disease in the UK:

a) Tuberculosis infection b) Autoimmune adrenalitis c) Adrenal carcinoma d) Waterhouse-Friderichsen syndrome e) Amyloidosis

Something wrong?

Primary insufciency of the adrenal cortex is called Addison’s disease, it is characterised by decient secretion of both glucocorticoids and mineralocorticoids.

Causes include:

autoimmune adrenalitis (most common cause in the UK) infection e.g. tuberculosis (most common cause of Addison’s disease worldwide, but is rare in the UK) tumour e.g. metastases amyloidosis haemorrhagic necrosis of the adrenal gland (e.g. Waterhouse-Friderichsen syndrome secondary to DIC in meningococcal septicaemia)

High levels of circulating ACTH can cause hyperpigmentation.

Features of Addison’s disease include:

postural hypotension hyponatraemia hyperkalaemia hypoglycaemia tendency for hypercalcaemia weight loss abdominal pain/constipation/nausea muscle weakness fatigue lethargy http://frcemsuccess.com/rev/sc­26/ 69/110 6/9/2017 Endocrine ­ FRCEM Success lethargy dizziness depression

An acute exacerbation of Addison’s disease is called an adrenal crisis. It is a life-threatening emergency characterised by: hypotensive shock, hypovolaemic shock and hypoglycaemia.

Acute adrenocortical failure may occur if long-term high-dose steroid treatment is stopped abruptly (as the prolonged steroid treatment has suppressed natural ACTH release). Secondary adrenocortical insufciency may also be caused by disorders of the hypothalamus and anterior pituitary gland due to deciency of CRH or ACTH.

Regarding parathyroid hormone (PTH), which of the following statements is CORRECT:

a) Parathyroid is synthesised by chief cells in the thyroid gland. b) PTH is released in response to rising serum Ca2+ levels. c) PTH release is inhibited by high serum phosphate. d) Low serum Mg+ may cause secondary hypoparathyroidism. e) PTH acts on the kidneys to increase reabsorption of calcium and phosphate ions.

Something wrong?

Parathyroid hormone (PTH) is synthesised by the chief cells in the parathyroid gland. PTH is released in response to falling plasma ionised calcium levels and increasing blood phosphate levels (indirectly by its binding to ionised calcium and thereby effective reduction of blood calcium levels).

PTH release is inhibited by normal blood calcium levels and hypomagnesaemia.

PTH acts to:

increase calcium and phosphate resorption from bone (via indirect upregulation of osteoclast activity) increase calcium reabsorption in the distal tubule of the nephron (by activating Ca2+ entry channels in the apical membrane and the Ca2+ ATPase pump on the basolateral membrane) increase phosphate excretion by inhibiting reabsorption in the proximal tubule of the nephron inhibit renal bicarbonate reabsorption stimulating a metabolism acidosis which favours dissociation of calcium from plasma proteins stimulate 1-alpha-hydroxylase in the kidneys to produce activated vitamin D indirectly increase calcium and phosphate absorption in the small intestine (via activated vitamin D)

Overall PTH acts to increase plasma calcium levels and decrease plasma phosphate levels.

Aldosterone release is directly stimulated by:

a) Angiotensin II b) Renin http://frcemcsu)ccAeDssH.c om/rev/sc­26/ 70/110 6/9/2017 Endocrine ­ FRCEM Success c) ADH d) Cortisol e) CRH

Something wrong?

Aldosterone is secreted by the zona glomerulosa of the adrenal cortex.

Aldosterone release is stimulated by:

angiotensin II a fall in extracellular uid volume (via the RAAS) a fall in plasma Na+ (via the RAAS) high plasma K+ ACTH

ACTH is less important as a regulator, so pituitary failure does not severely impair aldosterone secretion. Aldosterone acts mainly on the distal convoluted tubule (DCT) and the collecting duct of the kidney to cause increased reabsorption of sodium ions in exchange for potassium and hydrogen ions (via stimulation of Na+ pumps, Na+/H+ antiporters, and Na + and K+ channels in principal cells, and H+ ATPase in intercalated cells).

Water is also reabsorbed and blood volume therefore increased. Increased K+ excretion lowers plasma K+ levels. Aldosterone also stimulates Na+ conservation by the mucosal cells of the colon and gastric glands and by the sweat and salivary gland ducts.

What is the pathophysiology of Cushing’s disease:

a) Adrenocortical insufciency b) Catecholamine-secreting tumour c) Cortisol-secreting tumour d) ACTH-secreting tumour e) Aldosterone-secreting tumour

Something wrong?

Cushing’s disease refers to the specic condition of excess corticosteroids as a result of increased ACTH due to a pituitary adenoma; the negative feedback that normally prevents excess ACTH secretion is absent in the tumour. Cushing’s disease is associated with hyperpigmentation due to the melanocyte-stimulating action of ACTH.

Which of the following results would be expected in Graves’ disease:

a) Raised T3, Raised T4, Raised TSH http://frcemsuccess.com/rev/sc­26/ 71/110 6/9/2017 Endocrine ­ FRCEM Success

b) Raised T3, Raised T4, Low TSH c) Low T3, Low T4, Low TSH d) Low T3, Low T4, Raised TSH e) Raised T3, Raised T4, Normal TSH

Something wrong?

Graves’ disease is an autoimmune disease, in which autoantibodies against TSH receptors are produced. These antibodies bind to and stimulate these TSH receptors leading to an excess production of thyroid hormones. T3 and T4 levels are elevated, and TSH levels are low due to negative feedback.

Graves’ disease is the most common cause of hyperthyroidism.

Classically Grave’s disease is associated with:

hyperthyroidism pretibial myxoedema clubbing (thyroid acropachy) eye changes

Graves’ eye disease may include features of: exophthalmos, lid retraction, lid lag, diplopia, corneal ulcers.

Which of the following is NOT a well recognised feature of hyperglycaemia:

a) Polyuria b) Polydipsia c) Weight loss d) Palpitations e) Blurred vision

Something wrong?

Fasting levels of glucose are normally 3.9 – 5.5 mmol/L, random levels are normally < 11 mmol/L.

Glucose homeostasis is primarily maintained by the interplay between insulin and glucagon. Insulin is the only hormone that can lower glucose levels but adrenaline, cortisol and growth hormone, can raise levels in addition to glucagon (by stimulating glycogenolysis and gluconeogenesis).

Hyperglycaemia is dened as a fasting level of glucose > 7 mmol/L. (Levels < 6 mmol/L are normal, levels 6 – 6.9 mmol/L demonstrate impaired fasting glycaemia, and levels >= 7 mmol/L are diagnostic of diabetes). http://frcemsuccess.com/rev/sc­26/ 72/110 6/9/2017 Endocrine ­ FRCEM Success

Hyperglycaemia may occur in diabetes mellitus type 1 due to a deciency of insulin (due to autoimmune destruction of β-cells), in diabetes mellitus type 2 due to a insufciency of insulin and/or insulin resistance or due to secondary causes of diabetes.

Secondary causes of diabetes include:

Cushing’s syndrome thyrotoxicosis pheochromocytoma acromegaly glucagonoma cirrhosis chronic pancreatitis cystic brosis pancreatectomy haemochromatosis pancreatic carcinoma thiazide diuretics corticosteroid use.

Clinical features of hyperglycaemia include:

polyuria glycosuria dehydration polydipsia tendency to infections lethargy weight loss wasting blurred vision weakness

A patient who has been hospitalised with an intracranial bleed has become progressively more confused. He is found to be euvolaemic and normotensive with a poor urine output and hyponatraemia. What is the most likely diagnosis:

a) Acute renal failure b) SIADH c) Diabetes insipidus d) Hypothyroidism e) Conn’s syndrome

Something wrong?

http://frcemsuccess.com/rev/sc­26/ 73/110 6/9/2017 Endocrine ­ FRCEM Success

Syndrome of inappropriate ADH secretion (SIADH) causes water retention resulting in hypo-osmotic hyponatraemia.

Typical biochemistry shows:

hyponatraemia (< 125 mmol/L) low plasma osmolality (< 260 mmol/L) high levels of urinary sodium (> 20 mmol/L) high urine osmolality

Patients are euvolaemic and normotensive with normal thyroid and adrenal function.

The potential causes are vast including:

Neurological: tumour trauma infection Guillain-Barré syndrome multiple sclerosis systemic lupus erythematosus intracranial haemorrhage sinus thrombosis AIDS porphyria Pulmonary: small-cell lung cancer mesothelioma pneumonia abscess cystic brosis asthma tuberculosis positive-pressure ventilation Other malignancy: oropharyngeal stomach pancreas leukaemia lymphoma thymoma genitourinary tract cancers Drugs: chlorpropamide, carbamazepine, selective serotonin reuptake inhibitor (SSRI) antidepressants, tricyclic antidepressants, lithium, MDMA/ecstasy, tramadol, haloperidol, vincristine, desmopressin, uphenazine. Miscellaneous: idiopathic, hereditary, pain, postoperative, stress, endurance exercise and marathon runners, dermatomal herpes zoster. http://frcemsuccess.com/rev/sc­26/ 74/110 6/9/2017 Endocrine ­ FRCEM Success runners, dermatomal herpes zoster.

Regarding calcitonin, which of the following statements is CORRECT:

a) Calcitonin is secreted by parafollicular cells in the parathyroid gland. b) Calcitonin is secreted in response to rising blood phosphate levels. c) Calcitonin acts on the kidneys to inhibit reabsorption of calcium and phosphate. d) Calcitonin stimulates osteoclastic activity. e) Calcitonin acts on the small intestine to increase absorption of calcium.

Something wrong?

Calcitonin is secreted by the parafollicular cells in the thyroid gland. Calcitonin is secreted in response to rising or high blood calcium levels and acts to lower circulating levels of calcium.

Calcitonin acts on the kidneys to inhibit reabsorption of calcium and phosphate and on the bones to inhibit osteoclast activity and thus bone resorption of calcium and phosphate.

Which of the following is the main action of activated vitamin D:

a) Increased renal reabsorption of calcium b) Increased intestinal absorption of calcium and phosphate c) Increased renal reabsorption of phosphate d) Inhibition of a-alpha-hydroxylase e) Increased bone resorption

Something wrong?

Vitamin D (cholecalciferol) is absorbed by the small intestine as part of the diet or is synthesised from cholesterol in skin.

Cholecalciferol is converted to calcifediol by 25-hydroxylase in the liver. This is converted to activated vitamin D (1, 25 – dihydroxycholecalciferol or calcitriol) by 1-alpha-hydroxylase in the kidney. The enzyme 1-alpha-hydroxylase is stimulated by parathyroid hormone (PTH) and by low concentrations of phosphate.

Activated vitamin D acts to:

increase calcium and phosphate absorption in the small intestine (the main action) increase renal calcium reabsorption (in the distal tubule via activation of a basolateral Ca2+ ATPase pump) increase renal phosphate reabsorption inhibit 1-alpha-hydroxylase activity in the kidneys (negative feedback) affect bone remodelling to bring about the conditions of high calcium and phosphate optimum for bone remineralisation http://frcemsuccess.com/rev/sc­26/ 75/110 6/9/2017 Endocrine ­ FRCEM Success remineralisation

Vitamin D deciency can occur in:

dietary deciency malabsorption e.g. coeliac disease lack of sun exposure chronic kidney disease liver failure anticonvulsant therapy hypoparathyroidism

Deciency of vitamin D causes hypocalcaemia and osteomalacia (or rickets in children).

Aldosterone acts to cause which of the following:

a) Increased excretion of sodium ions b) Increased excretion of potassium ions c) Increased reabsorption of hydrogen ions d) Decreased plasma volume e) Decreased plasma osmolality

Something wrong?

Aldosterone is secreted by the zona glomerulosa of the adrenal cortex.

Aldosterone release is stimulated by:

angiotensin II a fall in extracellular uid volume (via the RAAS) a fall in plasma Na+ (via the RAAS) high plasma K+ ACTH

ACTH is less important as a regulator, so pituitary failure does not severely impair aldosterone secretion. Aldosterone acts mainly on the distal convoluted tubule (DCT) and the collecting duct of the kidney to cause increased reabsorption of sodium ions in exchange for potassium and hydrogen ions (via stimulation of Na+ pumps, Na+/H+ antiporters, and Na + and K+ channels in principal cells, and H+ ATPase in intercalated cells).

Water is also reabsorbed and blood volume therefore increased. Increased K+ excretion lowers plasma K+ levels. Aldosterone also stimulates Na+ conservation by the mucosal cells of the colon and gastric glands and by the sweat and salivary gland ducts.

Which of the following is NOT an action of cortisol: http://frcemsuccess.com/rev/sc­26/ 76/110 6/9/2017 Endocrine ­ FRCEM Success

a) Stimulates gluconeogenesis b) Increases proteolysis c) Stimulates lipolysis d) Increases calcium reabsorption in the kidneys e) Suppresses immune cells

Something wrong?

Cortisol is secreted from the zona fasciculata of the adrenal cortex. Cortisol release is stimulated by adrenocorticotrophic hormone (ACTH) from the anterior pituitary, which in turn is released in response to corticotrophin-releasing hormone (CRH) from the hypothalamus.

Cortisol has a negative feedback effect on the hypothalamus and the anterior pituitary gland. Cortisol release displays a circadian rhythm, with the highest levels in the early morning, peaking around 6 a.m. ACTH and thus cortisol release is stimulated by physical and psychological stressors (e.g. trauma, haemorrhage, fever).

Cortisol acts to:

raise blood glucose by stimulating gluconeogenesis and inhibiting peripheral glucose uptake increase protein breakdown in skeletal muscle, skin and bone to release amino acids stimulate lipolysis to increase fatty acid levels in the blood suppress the action and production of immune cells suppress the secretion of the anterior pituitary hormones weakly increase retention of sodium and water (a predominantly mineralocorticoid action) regulate calcium levels by decreasing calcium absorption from the gut increasing calcium excretion from the kidneys and increasing calcium resorption from bone inuence behaviour and cognitive function

What is the most common cause of secondary hyperaldosteronism:

a) Congenital adrenal hyperplasia b) Addison’s disease c) Ectopic ACTH secretion d) Excessive diuretic therapy e) Sudden withdrawal of long-term steroid therapy

Something wrong?

Hyperaldosteronism causes sodium and water retention (with volume expansion and hypertension) with increased excretion of potassium and hydrogen ions.

Features of hyperaldosteronism include: http://frcemsuccess.com/rev/sc­26/ 77/110 6/9/2017 Endocrine ­ FRCEM Success Features of hyperaldosteronism include:

hypertension hypokalaemia metabolic alkalosis polyuria and polydipsia muscle weakness and spasm N.B. Hypernatraemia rarely occurs due to other regulating mechanisms.

Conn’s syndrome is the most common cause of primary hyperaldosteronism, in which patients have an adenoma of the zona glomerulosa of the adrenal cortex.

In primary hyperaldosteronism, renin levels will be low due to negative feedback. High renin levels suggest secondary hyperaldosteronism e.g. secondary to excessive diuretic therapy (most common cause), congestive heart failure, renal artery stenosis, nephritic syndrome or cirrhosis with ascites.

Regarding hypopituitarism, which of the following statements is CORRECT:

a) The most common cause of hypopituitarism is primary pituitary failure. b) Sheehan’s syndrome refers to pituitary infarction following a stroke. c) Kallman’s syndrome is a congenital cause of hypogonadotropic hypogonadism. d) Hypopituitarism usually presents as an acute event with pituitary apoplexy. e) Pituitary adenomas cause excess hormone release, not hypopituitarism.

Something wrong?

Pituitary insufciency (hypopituitarism) often presents with insidious onset depression, tiredness and hypogonadism.

The most common cause of hypopituitarism is treatment of hyperpituitarism.

Other causes include:

compression of the pituitary by a non-functioning pituitary macroadenoma (most common type of pituitary tumour) compression of the pituitary by other local tumours e.g. craniopharyngioma, glioma, meningioma or metastases (particularly from the breast, bronchus and kidney) infarction of the pituitary gland e.g. Sheehan’s syndrome caused by hypovolaemic shock during obstetric haemorrhage empty sella syndrome inammatory/inltrative processes e.g. sarcoidosis haemochromatosis infective processes e.g. TB, syphilis pituitary hypoplasia or aplasia head injury congenital e.g. Kallman’s syndrome http://frcemsuccess.com/rev/sc­26/ 78/110 6/9/2017 Endocrine ­ FRCEM Success

Regarding vitamin D, which of the following statements is CORRECT:

a) Deciency of vitamin D may occur from lack of sun exposure. b) Severe vitamin D deciency in adults can cause rickets. c) Activated vitamin D causes increased Ca2+ reabsorption in the proximal tubule. d) Vitamin D activation is stimulated by calcitonin. e) Activated vitamin D inhibits bone remodelling.

Something wrong?

Vitamin D (cholecalciferol) is absorbed by the small intestine as part of the diet or is synthesised from cholesterol in skin.

Cholecalciferol is converted to calcifediol by 25-hydroxylase in the liver. This is converted to activated vitamin D (1, 25 – dihydroxycholecalciferol or calcitriol) by 1-alpha-hydroxylase in the kidney. The enzyme 1-alpha-hydroxylase is stimulated by parathyroid hormone (PTH) and by low concentrations of phosphate.

Activated vitamin D acts to:

increase calcium and phosphate absorption in the small intestine (the main action) increase renal calcium reabsorption (in the distal tubule via activation of a basolateral Ca2+ ATPase pump) increase renal phosphate reabsorption inhibit 1-alpha-hydroxylase activity in the kidneys (negative feedback) affect bone remodelling to bring about the conditions of high calcium and phosphate optimum for bone remineralisation

Vitamin D deciency can occur in:

dietary deciency malabsorption e.g. coeliac disease lack of sun exposure chronic kidney disease liver failure anticonvulsant therapy hypoparathyroidism

Deciency of vitamin D causes hypocalcaemia and osteomalacia (or rickets in children).

You are asked to assess a patient with hyponatraemia. They are hypovolaemic with a low plasma osmolality, and a urinary sodium of > 20 mmol/L, which of the following is the most likely cause of their hyponatraemia:

a) Poor oral intake due to dementia http://frcemsuccess.com/rev/sc­26/ 79/110 6/9/2017 Endocrine ­ FRCEM Success a) Poor oral intake due to dementia b) Fever c) Diabetes insipidus d) Addison’s disease e) SIADH

Something wrong?

Hyponatraemia is dened as a serum sodium concentration of below the reference interval of 133 – 146 mmol/L. A Na + concentration < 120 mmol/L is considered severe.

Hyponatraemia can arise either because of a primary loss of sodium ions (in excess of water loss) or more commonly, due to increased retention of water relative to sodium which effectively dilutes the concentration of sodium (both are associated with a decrease in plasma osmolality).

Patients with hyponatraemia can be divided into three categories on the basis of their ECF volume.

Hyponatraemia in hypovolaemic patients results from a true sodium decit (following loss of salt > water) and may be caused by:

Prerenal Losses (urinary Na+ < 20 mmol/L) Gastrointestinal loss e.g. diarrhoea, vomiting, stula Skin loss e.g. excessive sweating, dermatitis, burns Third space loss e.g. burns, pancreatitis, cirrhosis Renal losses (urinary Na+ > 20 mmol/L) diuretic therapy osmotic diuresis renal tubular disease mineralocorticoid deciency in Addison’s disease Poor dietary salt intake (very rare)

Hyponatraemia in euvolaemic patients results from excessive water retention either due to:

increased intake of pure water (where urine osmolality is low and urinary Na+ < 20 mmol/L) for example: psychogenic polydipsia inappropriate intravenous dextrose administration inability to excrete a water load (where urine osmolality is high and urinary Na+ > 20 mmol/L) which may be caused by: SIADH chronic renal disease severe hypothyroidism glucocorticoid deciency e.g. anterior pituitary disease or abrupt withdrawal of long-term glucocorticoids

Hyponatraemia in oedematous patients is usually associated with a high total body sodium, but there is a greater volume of water retained relative to sodium. Causes include: http://frcemsuccess.com/rev/sc­26/ 80/110 6/9/2017 Endocrine ­ FRCEM Success

Where urinary Na+ > 20 mmol/L renal failure cerebral salt wasting inappropriate intravenous hypertonic saline Where urinary Na+ < 20 mmol/L congestive cardiac failure malnutrition cirrhosis nephrotic syndrome

Which of the following features is NOT a typical feature of diabetes insipidus:

a) High plasma osmolality b) Hyponatraemia c) Polyuria d) Polydipsia e) Low urine osmolality

Something wrong?

Diabetes insipidus (DI) may result from a deciency of ADH secretion (cranial DI) or from an inappropriate renal response to ADH (nephrogenic DI).

As a result, uid reabsorption at the kidneys is impaired, resulting in large amounts of hypotonic, dilute urine being passed and subsequent polydipsia.

It is associated with elevated plasma osmolality (> 300 mOsm/kg) and low urine osmolality (< 600 mOsm/kg).

An ADH stimulation test can distinguish between cranial and nephrogenic DI, as nephrogenic DI shows an inability to concentrate urine even after administration of ADH.

Aldosterone release is directly stimulated by which of the following:

a) Increased plasma osmolality b) High plasma K+ c) Increased blood pressure d) Increased plasma volume e) Cortisol

Something wrong?

Aldosterone is secreted by the zona glomerulosa of the adrenal cortex. http://frcemsuccess.com/rev/sc­26/ 81/110 6/9/2017 Endocrine ­ FRCEM Success Aldosterone is secreted by the zona glomerulosa of the adrenal cortex.

Aldosterone release is stimulated by:

angiotensin II a fall in extracellular uid volume (via the RAAS) a fall in plasma Na+ (via the RAAS) high plasma K+ ACTH

ACTH is less important as a regulator, so pituitary failure does not severely impair aldosterone secretion. Aldosterone acts mainly on the distal convoluted tubule (DCT) and the collecting duct of the kidney to cause increased reabsorption of sodium ions in exchange for potassium and hydrogen ions (via stimulation of Na+ pumps, Na+/H+ antiporters, and Na + and K+ channels in principal cells, and H+ ATPase in intercalated cells).

Water is also reabsorbed and blood volume therefore increased. Increased K+ excretion lowers plasma K+ levels. Aldosterone also stimulates Na+ conservation by the mucosal cells of the colon and gastric glands and by the sweat and salivary gland ducts.

Which of the following is NOT a risk factor for osteoporosis:

a) Rheumatoid arthritis b) Alcohol c) Prolonged corticosteroid therapy d) Bisphosphonate therapy e) Smoking

Something wrong?

Osteoporosis is a disease characterised by low bone mineral density and deterioration of bone sufcient to cause bone fragility and an increased risk of fracture (particularly hip, spine and distal radius).

It is caused by increased osteoclast activity and decreased osteoblast activity resulting in a shift towards increased bone resorption.

Risk factors include:

post-menopausal women family history of osteoporotic fractures rheumatoid arthritis thyroid disease long-term corticosteroid use high alcohol intake smoking vitamin D deciency http://frcemsuccess.com/rev/sc­26/ 82/110 6/9/2017 Endocrine ­ FRCEM Success vitamin D deciency hyperparathyroidism

Which of the following is NOT a typical feature of hyperaldosteronism:

a) Hypertension b) Hypokalaemia c) Metabolic alkalosis d) Hyponatraemia e) Polyuria

Something wrong?

Hyperaldosteronism causes sodium and water retention (with volume expansion and hypertension) with increased excretion of potassium and hydrogen ions.

Features of hyperaldosteronism include:

hypertension hypokalaemia metabolic alkalosis polyuria and polydipsia muscle weakness and spasm N.B. Hypernatraemia rarely occurs due to other regulating mechanisms.

Conn’s syndrome is the most common cause of primary hyperaldosteronism, in which patients have an adenoma of the zona glomerulosa of the adrenal cortex.

In primary hyperaldosteronism, renin levels will be low due to negative feedback. High renin levels suggest secondary hyperaldosteronism e.g. secondary to excessive diuretic therapy (most common cause), congestive heart failure, renal artery stenosis, nephritic syndrome or cirrhosis with ascites.

Regarding the thyroid hormones, which of the following statements is CORRECT:

a) Thyrotropin-releasing hormone is secreted from the anterior pituitary gland. b) Iodine deciency results in excess thyroid hormone production. c) Thyroid hormones are secreted from parafollicular cells. d) T3 is converted to the more active T4 in the liver and kidney. e) Thyroid hormone secretion is stimulated by adrenaline.

Something wrong?

The release of thyroid hormones is regulated by the anterior pituitary gland which secretes thyroid-stimulating http://frcemsuccess.com/rev/sc­26/ 83/110 6/9/2017 Endocrine ­ FRCEM Success The release of thyroid hormones is regulated by the anterior pituitary gland which secretes thyroid-stimulating hormone (TSH) and the hypothalamus which secretes thyrotropin-releasing hormone (TRH).

The thyroid follicles secrete two hormones; thyroxine (T4) – a prohormone that acts as a plasma reservoir and tri- iodothyronine (T3) – the active hormone. T3 and T4 synthesis involves the processing of tyrosine and iodine.

About 90% of thyroid hormones are secreted in the form of T4, with the remainder as T3. About 80% of the T4 is converted to the more active T3 (under stimulation of TSH) in the liver and kidney.

Secretion of the thyroid hormones is stimulated by long-term exposure to cold temperatures acting on the anterior pituitary, oestrogens acting on the anterior pituitary and adrenaline acting directly on the thyroid gland. Increased serum levels of T3 inhibit secretion of TSH.

Most of the thyroid hormones in the blood are bound to plasma proteins (of these, 70% are bound to thyroid-binding globulin (TBG) and 30% are bound to albumin), which allows them to circulate without being broken down by enzymes.

Only a fraction of the circulating thyroid hormones (about 0.1% of T4 and 1% of T3) are unbound and thus biologically active. The thyroid hormones act to increase the basal metabolic rate and are important for growth and normal foetal development. Effects of thyroid hormones include: increased heart rate and stroke volume, and increased lipolysis, glycolysis and gluconeogenesis.

Which of the following would be considered a normal blood glucose level:

a) Random blood glucose of 3 mmol/L b) Random blood glucose of 12 mmol/L c) Fasting blood glucose of 5.5 mmol/L d) Fasting blood glucose 7.1 mmol/L e) Fasting blood glucose of 2.5 mmol/L

Something wrong?

Fasting levels of glucose are normally 3.9 – 5.5 mmol/L, random levels are normally < 11 mmol/L.

Glucose homeostasis is primarily maintained by the interplay between insulin and glucagon. Insulin is the only hormone that can lower glucose levels but adrenaline, cortisol and growth hormone, can raise levels in addition to glucagon (by stimulating glycogenolysis and gluconeogenesis).

Hyperglycaemia is dened as a fasting level of glucose > 7 mmol/L. (Levels < 6 mmol/L are normal, levels 6 – 6.9 mmol/L demonstrate impaired fasting glycaemia, and levels >/= 7 mmol/L are diagnostic of diabetes).

Hyperglycaemia may occur in diabetes mellitus type 1 due to a deciency of insulin (due to autoimmune destruction of β-cells), in diabetes mellitus type 2 due to a insufciency of insulin and/or insulin resistance or due to secondary causes of diabetes.

Secondary causes of diabetes include: http://frcemsuccess.com/rev/sc­26/ 84/110 6/9/2017 Endocrine ­ FRCEM Success Secondary causes of diabetes include:

Cushing’s syndrome thyrotoxicosis pheochromocytoma acromegaly glucagonoma cirrhosis chronic pancreatitis cystic brosis pancreatectomy haemochromatosis pancreatic carcinoma thiazide diuretics corticosteroid use.

Clinical features of hyperglycaemia include:

polyuria glycosuria dehydration polydipsia tendency to infections lethargy weight loss wasting blurred vision weakness

Regarding growth hormone, which of the following statements is INCORRECT:

a) Growth hormone stimulates the production of insulin-like growth factor. b) Growth hormone acts to increase blood glucose levels. c) Growth hormone release is stimulated by thyroid hormones. d) Excess levels of growth hormone in young children results in acromegaly. e) Growth hormone is secreted from the anterior pituitary gland.

Something wrong?

Growth hormone is secreted from the anterior pituitary gland.

Growth hormone (GH) acts on the liver to stimulate insulin-like growth factor (IGF) production and to oppose the actions of insulin (amongst other effects).

GH secretion is stimulated by growth hormone-releasing hormone (GHRH) from the hypothalamus e.g. in response to http://frcemhsuycpcoegssly.ccoame/mreiva/s acn­2d6 /by thyroid hormones from the thyroid gland. GH secretion is inhibited by growth hormone- 85/110 6/9/2017 Endocrine ­ FRCEM Success hypoglycaemia and by thyroid hormones from the thyroid gland. GH secretion is inhibited by growth hormone- inhibiting hormone (or somatostatin) and IGF-1.

Excess levels of GH e.g. due to a functioning pituitary adenoma results in acromegaly in adults and gigantism in children (if excess GH occurs prior to epiphyseal fusion). GH deciency results in dwarsm in children or adult GH deciency syndrome in adults.

Glucagon secretion is inhibited by all BUT which one of the following:

a) Fatty acids and ketones b) Insulin c) Somatostatin d) High blood glucose e) Adrenaline

Something wrong?

Glucagon is synthesised in the pancreatic alpha (α) cells in the islets of Langerhans.

Glucagon secretion is stimulated by low blood glucose, amino acids, adrenaline, some gastrointestinal peptides, sympathetic and parasympathetic innervation. Glucagon secretion is inhibited by high blood glucose, fatty acids and ketones, insulin and somatostatin.

Glucagon acts on G-protein coupled receptors.

Glucagon acts to:

stimulate glycogenolysis inhibit peripheral tissue glucose uptake inhibit glycolysis inhibit amino acid uptake and protein synthesis stimulate lipolysis stimulate gluconeogenesis stimulate ketogenesis

Somatostatin is secreted by which of the following pancreatic cell types:

a) α cells b) β cells c) δ cells d) F-cells e) Acinar cells

Something wrong? http://frcemsuccess.com/rev/sc­26/ 86/110 6/9/2017 Endocrine ­ FRCEM Success

The endocrine cells of the pancreas are arranged in small clusters around the larger exocrine cell clusters, called acini.

The endocrine clusters are called islets of Langerhans and within them are four types of cells:

1. alpha cells (comprise 20% of all cells) secrete glucagon 2. beta cells (70%) secrete insulin 3. delta cells (8%) secrete somatostatin 4. F-cells ( 2%) secrete pancreatic polypeptide.

Insulin and glucagon regulate blood glucose levels, somatostatin inhibits the release of both insulin and glucagon, pancreatic polypeptide inhibits the exocrine functions of the pancreas.

Regarding general management of hyponatraemia, which of the following statements is CORRECT:

a) Normovolaemic patients with hyponatraemia usually require replacement of sodium in the form of isotonic saline. b) Rapid correction of hyponatraemia is associated with central pontine myelinolysis. c) Neurological dysfunction resulting from hyponatraemia reects crenation of cerebral cells. d) Seizures and focal neurological signs are commonly seen in hyponatraemia of 120 mmol/L or less. e) Hypovolaemic patients with hyponatraemia usually require replacement of sodium in the form of hypertonic saline.

Something wrong?

Symptoms due to hyponatraemia reect neurological dysfunction resulting from cerebral overhydration (with progressive cell swelling and cytolysis) induced by hypo-osmolality.

They are non-specic symptoms and include nausea, malaise, headache, lethargy, muscle cramps and weakness, ataxia and reduced level of consciousness. Seizures, comas and focal neurological signs are not usually seen until the sodium concentration is less than about 115 mmol/L.

In cases of hyponatraemia due to sodium depletion, there are usually signs of volume depletion. In cases of hyponatraemia due to uid retention, patients may be euvolaemic or uid overloaded.

Regarding management in general:

Hypovolaemic patients are sodium-depleted and should be given sodium in the form of isotonic saline initially. Normovolaemic patients are likely to have normal total body sodium but are retaining uid and should be uid restricted. Oedematous patient have an excess of total body sodium and water, and should be given a diuretic to induce natriuresis and be uid restricted. http://frcemsuccess.com/rev/sc­26/ 87/110 6/9/2017 Endocrine ­ FRCEM Success natriuresis and be uid restricted.

In acute symptomatic hyponatraemia, the use of hypertonic saline should be considered with specialist input. Correction should not be too rapid, especially in chronic hyponatraemia, as this is may result in central pontine myelinolysis.

Which hormone does not function appropriately in diabetes insipidus:

a) Aldosterone b) Angiotensin II c) Renin d) Antidiuretic hormone e) Insulin

Something wrong?

Diabetes insipidus (DI) may result from a deciency of ADH secretion (cranial DI) or from an inappropriate renal response to ADH (nephrogenic DI).

As a result, uid reabsorption at the kidneys is impaired, resulting in large amounts of hypotonic, dilute urine being passed and subsequent polydipsia.

It is associated with elevated plasma osmolality (> 300 mOsm/kg) and low urine osmolality (< 600 mOsm/kg).

An ADH stimulation test can distinguish between cranial and nephrogenic DI, as nephrogenic DI shows an inability to concentrate urine even after administration of ADH.

How does ADH increase water reabsorption:

a) Insertion of aquaporins b) Stimulates release of renin c) Upregulation of Na+/K+ ATPase d) Decreases GFR by afferent arteriole vasoconstriction e) Upregulation of Na+ channels

Something wrong?

Antidiuretic hormone is synthesised in the hypothalamus and stored in secretory granules in the posterior pituitary. The main action on the kidneys is to increase the permeability of the distal tubule and the collecting duct to reabsorb water and thus increase water retention and concentrate urine.

ADH binds to V2 receptors on renal principal cells and increases cAMP, causing the incorporation of water channels http://frcemcsaucllceeds sa.cqouma/preovr/sincs­2 i6n/to the apical membrane. It also has a potent vasoconstriction action at high doses. 88/110 6/9/2017 Endocrine ­ FRCEM Success called aquaporins into the apical membrane. It also has a potent vasoconstriction action at high doses.

ADH is rapidly removed from plasma, falling by about 50% in about 10 minutes, mainly due to metabolism in the liver and kidneys.

ADH release is stimulated by raised plasma osmolality (detected by osmoreceptors in the hypothalamus which also stimulate thirst) and a fall in blood pressure/plasma volume (detected by cardiac and vascular baroreceptors). The osmoreceptor system is more sensitive than the baroreceptor system.

ADH release is inhibited by low plasma osmolality, alcohol, caffeine, glucocorticoids and atrial natriuretic peptide (ANP).

ADH deciency (or an inadequate response to ADH) results in diabetes insipidus. Excess levels of ADH results in syndrome of inappropriate ADH secretion (SIADH).

Which of the following is NOT a cause of hyperprolactinaemia:

a) Pituitary adenoma b) Hyperthyroidism c) Polycystic ovary syndrome d) Domperidone e) Pregnancy

Something wrong?

Prolactin acts on the mammary glands and reproductive organs to promote growth of these organs and initiate lactation.

Prolactin secretion is stimulated by prolactin-releasing factor (PRF) and thyrotropin-releasing hormone (TRH) from the hypothalamus. Prolactin secretion is inhibited by dopamine secreted by the hypothalamus.

Prolactin levels rise physiologically in pregnancy, puerperium, and breast stimulation.

Excess levels of prolactin may be caused by:

a prolactinoma (prolactin-secreting pituitary adenoma) compression of the pituitary stalk by a pituitary or hypothalamic tumour preventing normal dopaminergic inhibition of prolactin release head injury neurosurgery hypothyroidism (due to increased synthesis of TRH) Cushing’s syndrome severe liver disease polycystic ovary syndrome (PCOS) drugs e.g. dopamine antagonists, antidepressants. http://frcemsuccess.com/rev/sc­26/ 89/110 6/9/2017 Endocrine ­ FRCEM Success drugs e.g. dopamine antagonists, antidepressants.

Hyperprolactinaemia causes symptoms such as oligomenorrhoea/amenorrhoea, galactorrhoea, loss of libido, erectile dysfunction and infertility (via inhibition of the release of GnRH from the hypothalamus).

Prolactin deciency results in failure of postpartum lactation.

Antidiuretic hormone acts primarily at which of the following locations in the renal nephron:

a) Proximal tubule b) Thick ascending limb of loop of Henle c) Thin ascending limb of loop of Henle d) Descending limb of loop of Henle e) Collecting ducts

Something wrong?

Antidiuretic hormone is synthesised in the hypothalamus and stored in secretory granules in the posterior pituitary. The main action on the kidneys is to increase the permeability of the distal tubule and the collecting duct to reabsorb water and thus increase water retention and concentrate urine.

ADH binds to V2 receptors on renal principal cells and increases cAMP, causing the incorporation of water channels called aquaporins into the apical membrane. It also has a potent vasoconstriction action at high doses.

ADH is rapidly removed from plasma, falling by about 50% in about 10 minutes, mainly due to metabolism in the liver and kidneys.

ADH release is stimulated by raised plasma osmolality (detected by osmoreceptors in the hypothalamus which also stimulate thirst) and a fall in blood pressure/plasma volume (detected by cardiac and vascular baroreceptors). The osmoreceptor system is more sensitive than the baroreceptor system.

ADH release is inhibited by low plasma osmolality, alcohol, caffeine, glucocorticoids and atrial natriuretic peptide (ANP).

ADH deciency (or an inadequate response to ADH) results in diabetes insipidus. Excess levels of ADH results in syndrome of inappropriate ADH secretion (SIADH).

Which of the following signs would you NOT expect to see in hyperthyroidism:

a) Hyporeexia b) Palmar erythema c) Clubbing d) Onycholysis e) Irregularly irregular pulse http://frcemsuccess.com/rev/sc­26/ 90/110 6/9/2017 Endocrine ­ FRCEM Success

Something wrong?

Hyperthyroidism is dened as over activity of the thyroid gland, leading to excess thyroid hormone secretion.

Causes include:

Graves’ disease toxic multinodular goitre toxic adenoma ectopic thyroid tissue drugs e.g. amiodarone exogenous iodine

Features of hyperthyroidism include:

hair loss emotional lability fatigue anxiety restlessness goitre palpitations tachycardia atrial brillation proximal myopathy diarrhoea increased appetite weight loss tremor heat intolerance sweating oligomenorrhoea/amenorrhoea infertility reduced libido osteoporosis hyperreexia pretibial myxoedema (in Graves’ disease) palmar erythema clubbing onycholysis

T3 and T4 levels are elevated, and TSH levels are usually low. A raised TSH suggests the fault lies in or above the pituitary gland.

http://frcemsuccess.com/rev/sc­26/ 91/110 6/9/2017 Endocrine ­ FRCEM Success

Regarding the thyroid hormones, which of the following statements is CORRECT:

a) Thyroxine (T4) is more active than tri-iodothyronine (T3). b) Thyroid hormones are predominantly secreted in the form of T3. c) Increased serum levels of T3 inhibits secretion of TSH. d) Thyroid hormones are released in response to TSH, secreted by the hypothalamus. e) About 50% of thyroid hormones in the blood are bound to plasma proteins.

Something wrong?

The release of thyroid hormones is regulated by the anterior pituitary gland which secretes thyroid-stimulating hormone (TSH) and the hypothalamus which secretes thyrotropin-releasing hormone (TRH).

The thyroid follicles secrete two hormones; thyroxine (T4) – a prohormone that acts as a plasma reservoir and tri- iodothyronine (T3) – the active hormone. T3 and T4 synthesis involves the processing of tyrosine and iodine.

About 90% of thyroid hormones are secreted in the form of T4, with the remainder as T3. About 80% of the T4 is converted to the more active T3 (under stimulation of TSH) in the liver and kidney.

Secretion of the thyroid hormones is stimulated by long-term exposure to cold temperatures acting on the anterior pituitary, oestrogens acting on the anterior pituitary and adrenaline acting directly on the thyroid gland. Increased serum levels of T3 inhibit secretion of TSH.

Most of the thyroid hormones in the blood are bound to plasma proteins (of these, 70% are bound to thyroid-binding globulin (TBG) and 30% are bound to albumin), which allows them to circulate without being broken down by enzymes.

Only a fraction of the circulating thyroid hormones (about 0.1% of T4 and 1% of T3) are unbound and thus biologically active. The thyroid hormones act to increase the basal metabolic rate and are important for growth and normal foetal development. Effects of thyroid hormones include: increased heart rate and stroke volume, and increased lipolysis, glycolysis and gluconeogenesis.

What is the most likely diagnosis in a 70 year old woman with rheumatoid arthritis who presents with weight gain, moon face and resistant hypertension:

a) Cushing’s syndrome b) Cushing’s disease c) Pheochromocytoma d) Conn’s syndrome e) Renal artery stenosis

Something wrong?

Cushing’s syndrome is a rare condition caused by a chronic excess of glucocorticoids. http://frcemsuccess.com/rev/sc­26/ 92/110 6/9/2017 Endocrine ­ FRCEM Success Cushing’s syndrome is a rare condition caused by a chronic excess of glucocorticoids.

This may occur due to:

1. excess secretion of ACTH or CRH (e.g. ACTH secreting pituitary adenoma, ectopic ACTH secretion by small- cell carcinoma of the lung or bronchial carcinoid tumour) 2. excess secretion of cortisol (e.g. adenoma of the adrenal cortex (most common cause of Cushing’s syndrome in children) or adrenal carcinoma) 3. exogenous steroids (most common cause of Cushing’s syndrome in adults).

Features of Cushing’s syndrome include:

hair thinning male-pattern baldness cataracts truncal obesity striae dorsocervical fat pad (buffalo-hump) moon face acne hirsutism hypertension skeletal muscle weakness and wasting peptic ulceration renal calculi menstrual disturbance osteoporosis and associated increased risk of fracture/vertebral collapse skin thinning, easy bruising, tendency to skin infections ankle oedema glucose intolerance/DM predisposition to congestive cardiac failure depression/confusion/insomnia/psychosis

Which of the following statements regarding the pituitary gland is CORRECT:

a) The anterior pituitary is really a direct extension of the hypothalamus. b) Hypothalamic hormones are transported to the posterior pituitary by hypophyseal portal vessels. c) Dopamine released from the hypothalamus stimulates the release of prolactin. d) Oxytocin release from the posterior pituitary is controlled primarily by negative feedback mechanisms. e) Cortisol inhibits release of adrenocorticotrophic hormone (ACTH) from the anterior pituitary.

Something wrong?

Anterior pituitary hormones are released under the control of hypothalamic releasing or inhibiting hormones originating from small neurones with their cell bodies in the hypothalamus. http://frcemsuccess.com/rev/sc­26/ 93/110 6/9/2017 Endocrine ­ FRCEM Success originating from small neurones with their cell bodies in the hypothalamus.

These hypothalamic hormones are transported directly to the anterior pituitary via hypophyseal portal vessels to stimulate or inhibit release of anterior pituitary hormones e.g. corticotrophin-releasing hormone (CRH) stimulates release of ACTH, thyrotropin-releasing hormone stimulates release of TSH (and prolactin) and dopamine inhibits release of prolactin.

The anterior pituitary hormones (and the hormones released by their target organs) inhibit further release of hypothalamic and anterior pituitary hormones by negative feedback mechanisms.

The posterior pituitary is really a direct extension of the hypothalamus. Oxytocin and ADH are manufactured in the cell bodies of neurones in the hypothalamus and are transported down the axons of these cells to their terminals on capillaries originating from the inferior hypophyseal artery within the posterior pituitary gland.

When these neurones are activated, they release oxytocin or ADH into the general circulation. ADH release is controlled by negative feedback mechanisms, oxytocin however is involved in positive feedback mechanisms.

Which of the following is a cause of hyperthyroidism:

a) Hypopituitarism b) Hashimoto’s thyroiditis c) Iodine deciency d) Carbimazole e) Amiodarone

Something wrong?

Hyperthyroidism is dened as over activity of the thyroid gland, leading to excess thyroid hormone secretion.

Causes include:

Graves’ disease toxic multinodular goitre toxic adenoma ectopic thyroid tissue drugs e.g. amiodarone, lithium rarely exogenous iodine

Features of hyperthyroidism include:

hair loss emotional lability fatigue anxiety restlessness http://frcemsuccess.com/rev/sc­26/ 94/110 6/9/2017 Endocrine ­ FRCEM Success

goitre palpitations tachycardia atrial brillation proximal myopathy diarrhoea increased appetite weight loss tremor heat intolerance sweating oligomenorrhoea/amenorrhoea infertility reduced libido osteoporosis hyperreexia pretibial myxoedema (in Graves’ disease) palmar erythema clubbing onycholysis

T3 and T4 levels are elevated, and TSH levels are usually low. A raised TSH suggests the fault lies in or above the pituitary gland.

Insulin secretion is inhibited by all but which one of the following:

a) Adrenaline b) Sympathetic innervation c) Low blood glucose d) Somatostatin e) Secretin

Something wrong?

Insulin is a polypeptide hormone consisting of two short chains (A and B) linked by disulphide bonds.

Insulin is formed from the cleaving of proinsulin (derived from preproinsulin synthesised in the rough endoplasmic reticulum) into insulin and C-peptide in the Golgi body of β cells in the islets of Langerhans.

Since insulin and C-peptide are produced in equimolar amounts, C-peptide acts as a useful marker of β cell activity in diabetics who receive insulin treatment.

Insulin secretion is stimulated directly by high blood glucose levels, but also by metabolites such as amino acids, fatty acids and ketones, by glucagon, some gastrointestinal tract peptides (e.g. secretin), GH, ACTH and TSH. Insulin http://frcemseuccrestsio.cnom is/ rienvh/sibc­i2t6e/d by low blood glucose levels, adrenaline, somatostatin, hypocalcaemia and sympathetic 95/110 6/9/2017 Endocrine ­ FRCEM Success secretion is inhibited by low blood glucose levels, adrenaline, somatostatin, hypocalcaemia and sympathetic innervation. (N.B. insulin secretion never ceases completely, there is always a basal level of insulin in the blood)

What is the most likely diagnosis in a 65 year old patient presenting with polyuria, polydipsia, renal failure and deteriorating sight:

a) Renal artery stenosis b) Diabetes mellitus c) Acromegaly d) Diabetes insipidus e) Cushing’s syndrome

Something wrong?

Hyperglycaemia is dened as a fasting level of glucose > 7 mmol/L.

It can occur in diabetes mellitus type 1 due to a deciency of insulin (due to autoimmune destruction of beta-cells) or in diabetes mellitus type 2 due to a insufciency of insulin and/or insulin resistance or in secondary causes of diabetes.

Secondary causes of diabetes include:

Cushing’s syndrome thyrotoxicosis pheochromocytoma acromegaly glucagonoma cirrhosis chronic pancreatitis cystic brosis pancreatectomy haemochromatosis pancreatic carcinoma thiazide diuretics corticosteroid use

Features of hyperglycaemia include polyuria, glycosuria, dehydration, polydipsia, tendency to infections, lethargy, weight loss, wasting, blurred vision, weakness.

Long-term complications of hyperglycaemia in diabetes mellitus include: macrovascular complications (hyperlipidaemia, stroke, ischaemic heart disease, renal artery stenosis, hypertension, peripheral vascular disease) and microvascular complications (lacunar infarcts, diabetic retinopathy, cataracts, glaucoma, diabetic nephropathy and peripheral neuropathy).

Regarding osteoporosis, which of the following statements is CORRECT: http://frcemsuccess.com/rev/sc­26/ 96/110 6/9/2017 Regarding osteoporosis, which of the Efondlolocrwinei n­ gFR sCtaEMte Smucecnestss is CORRECT:

a) Osteoporosis occurs as a direct result of vitamin D deciency. b) Osteoporosis increases risk of fracture especially of the proximal arm. c) Osteoporosis is characterised by increased osteoclastic activity. d) Alcohol is actually protective against osteoporosis. e) Osteoporosis is a disorder of defective mineralisation of bone matrix.

Something wrong?

Osteoporosis is a disease characterised by low bone mineral density and deterioration of bone sufcient to cause bone fragility and an increased risk of fracture (particularly hip, spine and distal radius).

It is caused by increased osteoclast activity and decreased osteoblast activity resulting in a shift towards increased bone resorption.

Risk factors include:

post-menopausal women family history of osteoporotic fractures rheumatoid arthritis thyroid disease long-term corticosteroid use high alcohol intake smoking vitamin D deciency hyperparathyroidism

Regarding diabetes mellitus (DM), which of the following statements is CORRECT:

a) Type 1 diabetes mellitus results from insulin resistance. b) A fasting blood glucose > 6 mmol/L is diagnostic of DM. c) Chronic pancreatitis can cause a secondary diabetes. d) Blood glucose levels may be raised by cortisol deciency. e) Hyperglycaemia typically results in large volumes of highly dilute urine.

Something wrong?

Fasting levels of glucose are normally 3.9 – 5.5 mmol/L, random levels are normally < 11 mmol/L.

Glucose homeostasis is primarily maintained by the interplay between insulin and glucagon. Insulin is the only hormone that can lower glucose levels but adrenaline, cortisol and growth hormone, can raise levels in addition to glucagon (by stimulating glycogenolysis and gluconeogenesis). http://frcemsuccess.com/rev/sc­26/ 97/110 6/9/2017 Endocrine ­ FRCEM Success

Hyperglycaemia is dened as a fasting level of glucose > 7 mmol/L. (Levels < 6 mmol/L are normal, levels 6 – 6.9 mmol/L demonstrate impaired fasting glycaemia, and levels >= 7 mmol/L are diagnostic of diabetes).

Hyperglycaemia may occur in diabetes mellitus type 1 due to a deciency of insulin (due to autoimmune destruction of β-cells), in diabetes mellitus type 2 due to a insufciency of insulin and/or insulin resistance or due to secondary causes of diabetes.

Secondary causes of diabetes include:

Cushing’s syndrome thyrotoxicosis pheochromocytoma acromegaly glucagonoma cirrhosis chronic pancreatitis cystic brosis pancreatectomy haemochromatosis pancreatic carcinoma thiazide diuretics corticosteroid use

Clinical features of hyperglycaemia include:

polyuria glycosuria dehydration polydipsia tendency to infections lethargy weight loss wasting blurred vision weakness

Which of the following is NOT a typical feature of Cushing’s syndrome:

a) Male-pattern baldness b) Osteoporosis c) Cataracts d) Pancreatitis e) Glucose intolerance

Something wrong?

http://frcemsuccess.com/rev/sc­26/ 98/110 6/9/2017 Endocrine ­ FRCEM Success

Cushing’s syndrome is a rare condition caused by a chronic excess of glucocorticoids.

This may occur due to:

1. excess secretion of ACTH or CRH (e.g. ACTH secreting pituitary adenoma, ectopic ACTH secretion by small- cell carcinoma of the lung or bronchial carcinoid tumour) 2. excess secretion of cortisol (e.g. adenoma of the adrenal cortex (most common cause of Cushing’s syndrome in children) or adrenal carcinoma) 3. exogenous steroids (most common cause of Cushing’s syndrome in adults).

Features of Cushing’s syndrome include:

hair thinning male-pattern baldness cataracts truncal obesity striae dorsocervical fat pad (buffalo-hump) moon face acne hirsutism hypertension skeletal muscle weakness and wasting peptic ulceration renal calculi menstrual disturbance osteoporosis and associated increased risk of fracture/vertebral collapse skin thinning, easy bruising, tendency to skin infections ankle oedema glucose intolerance/DM predisposition to congestive cardiac failure depression/confusion/insomnia/psychosis

Dehydroepiandrosterone (DHEA) is secreted by which of the following:

a) Zona glomerulosa of the adrenal cortex b) Zona reticularis of the adrenal cortex c) Zona fasciculata of the adrenal cortex d) Adrenal medulla e) Anterior pituitary

Something wrong?

http://frcemsuccess.com/rev/sc­26/ 99/110 6/9/2017 Endocrine ­ FRCEM Success

Adrenal androgens are secreted from the zona reticularis of the adrenal cortex. The main adrenal androgens are dehydroepiandrosterone (DHEA) and androstenedione. Adrenal androgens have only weak biological activity but they are converted to more active androgens, such as testosterone, by aromatase and other enzymes in peripheral tissues.

Aldosterone primarily acts at which of the following sites in the renal nephron:

a) Proximal tubule b) Descending limb c) Thick ascending limb d) Thin ascending limb e) Distal convoluted tubule and collecting ducts

Something wrong?

Aldosterone is secreted by the zona glomerulosa of the adrenal cortex.

Aldosterone release is stimulated by:

angiotensin II a fall in extracellular uid volume (via the RAAS) a fall in plasma Na+ (via the RAAS) high plasma K+ ACTH

ACTH is less important as a regulator, so pituitary failure does not severely impair aldosterone secretion.

Aldosterone acts mainly on the distal convoluted tubule (DCT) and the collecting duct of the kidney to cause increased reabsorption of sodium ions in exchange for potassium and hydrogen ions (via stimulation of Na+ pumps, Na+/H+ antiporters, and Na+ and K+ channels in principal cells, and H+ ATPase in intercalated cells).

Water is also reabsorbed and blood volume therefore increased. Increased K+ excretion lowers plasma K+ levels. Aldosterone also stimulates Na+ conservation by the mucosal cells of the colon and gastric glands and by the sweat and salivary gland ducts.

Corticotrophin-releasing hormone stimulates release of which of the following pituitary hormones:

a) ACTH b) ADH c) GH d) TSH e) Prolactin http://frcemsuccess.com/rev/sc­26/ 100/110 6/9/2017 Endocrine ­ FRCEM Success

Something wrong?

Anterior pituitary hormones are released under the control of hypothalamic releasing or inhibiting hormones originating from small neurones with their cell bodies in the hypothalamus.

These hypothalamic hormones are transported directly to the anterior pituitary via hypophyseal portal vessels to stimulate or inhibit release of anterior pituitary hormones e.g. corticotrophin-releasing hormone (CRH) stimulates release of ACTH, thyrotropin-releasing hormone stimulates release of TSH (and prolactin) and dopamine inhibits release of prolactin.

The anterior pituitary hormones (and the hormones released by their target organs) inhibit further release of hypothalamic and anterior pituitary hormones by negative feedback mechanisms.

The posterior pituitary is really a direct extension of the hypothalamus. Oxytocin and ADH are manufactured in the cell bodies of neurones in the hypothalamus and are transported down the axons of these cells to their terminals on capillaries originating from the inferior hypophyseal artery within the posterior pituitary gland.

When these neurones are activated, they release oxytocin or ADH into the general circulation. ADH release is controlled by negative feedback mechanisms, oxytocin however is involved in positive feedback mechanisms.

What is the most likely diagnosis in an otherwise healthy 25 year old woman who presents with polyuria and polydipsia, with no history of weight loss. She has a past medical history of a traumatic brain injury:

a) Diabetes mellitus type 1 b) Diabetes mellitus type 2 c) SIADH d) Cranial diabetes insipidus e) Nephrogenic diabetes insipidus

Something wrong?

Diabetes insipidus (DI) may result from a deciency of ADH secretion (cranial DI) or from an inappropriate renal response to ADH (nephrogenic DI). Cranial DI may be the result of hypoxic encephalopathy, neurosurgery, autoimmunity or cancer, or sometimes without an underlying cause (idiopathic).

As a result, uid reabsorption at the kidneys is impaired, resulting in large amounts of hypotonic, dilute urine being passed and subsequent polydipsia.

It is associated with elevated plasma osmolality (> 300 mOsm/kg) and low urine osmolality (< 600 mOsm/kg).

An ADH stimulation test can distinguish between cranial and nephrogenic DI, as nephrogenic DI shows an inability to http://frcemsuccess.com/rev/sc­26/ 101/110 6/9/2017 Endocrine ­ FRCEM Success

concentrate urine even after administration of ADH.

Which of the following is the most common cause of Cushing’s syndrome in children:

a) ACTH-secreting pituitary adenoma b) Ectopic ACTH secreting small-cell carcinoma of the lung c) Adrenal adenoma d) Adrenal carcinoma e) Exogenous glucocorticoids

Something wrong?

Cushing’s syndrome is a rare condition caused by a chronic excess of glucocorticoids.

This may occur due to:

1. excess secretion of ACTH or CRH (e.g. ACTH secreting pituitary adenoma, ectopic ACTH secretion by small- cell carcinoma of the lung or bronchial carcinoid tumour) 2. excess secretion of cortisol (e.g. adenoma of the adrenal cortex (most common cause of Cushing’s syndrome in children) or adrenal carcinoma) 3. exogenous steroids (most common cause of Cushing’s syndrome in adults).

Features of Cushing’s syndrome include:

hair thinning male-pattern baldness cataracts truncal obesity striae dorsocervical fat pad (buffalo-hump) moon face acne hirsutism hypertension skeletal muscle weakness and wasting peptic ulceration renal calculi menstrual disturbance osteoporosis and associated increased risk of fracture/vertebral collapse skin thinning, easy bruising, tendency to skin infections ankle oedema glucose intolerance/DM predisposition to congestive cardiac failure depression/confusion/insomnia/psychosis http://frcemsuccess.com/rev/sc­26/ 102/110 6/9/2017 Endocrine ­ FRCEM Success

Which of the following is NOT an effect of parathyroid hormone (PTH):

a) Increases Ca2+ reabsorption in the distal tubule 3- b) Increases PO4 reabsorption in the proximal tubule c) Increases bone resorption d) Stimulates activation of vitamin D by the kidneys – e) Inhibits renal HCO3 reabsorption

Something wrong?

Parathyroid hormone (PTH) is synthesised by the chief cells in the parathyroid gland. PTH is released in response to falling plasma ionised calcium levels and increasing blood phosphate levels (indirectly by its binding to ionised calcium and thereby effective reduction of blood calcium levels).

PTH release is inhibited by normal blood calcium levels and hypomagnesaemia.

PTH acts to:

increase calcium and phosphate resorption from bone (via indirect upregulation of osteoclast activity) increase calcium reabsorption in the distal tubule of the nephron (by activating Ca2+ entry channels in the apical membrane and the Ca2+ ATPase pump on the basolateral membrane) increase phosphate excretion by inhibiting reabsorption in the proximal tubule of the nephron inhibit renal bicarbonate reabsorption stimulating a metabolism acidosis which favours dissociation of calcium from plasma proteins stimulate 1-alpha-hydroxylase in the kidneys to produce activated vitamin D indirectly increase calcium and phosphate absorption in the small intestine (via activated vitamin D)

Overall PTH acts to increase plasma calcium levels and decrease plasma phosphate levels.

What is the most likely diagnosis in a 55 year old woman presenting with atrial brillation, tremor and exophthalmos:

a) Acromegaly b) Cushing’s disease c) Graves’ disease d) Hashimoto thyroiditis e) Pheochromocytoma

Something wrong?

Graves’ disease is an autoimmune disease, in which autoantibodies against TSH receptors are produced. These http://frcemsuccess.com/rev/sc­26/ 103/110 6/9/2017 Endocrine ­ FRCEM Success Graves’ disease is an autoimmune disease, in which autoantibodies against TSH receptors are produced. These antibodies bind to and stimulate these TSH receptors leading to an excess production of thyroid hormones. Graves’ disease is the most common cause of hyperthyroidism.

Classically Grave’s disease is associated with:

hyperthyroidism pretibial myxoedema clubbing (thyroid acropachy) eye changes including features of: exophthalmos, lid retraction, lid lag, diplopia, corneal ulcers

Insulin acts to cause all but which one of the following effects:

a) Stimulation of glycogenesis b) Stimulation of glycolysis c) Inhibition of proteolysis d) Stimulation of lipolysis e) Inhibition of gluconeogenesis

Something wrong?

Insulin acts to:

stimulate peripheral tissue glucose uptake stimulate glycogenesis stimulate glycolysis stimulate amino acid uptake and protein synthesis inhibit glycogenolysis inhibit proteolysis stimulate lipogenesis inhibit lipolysis inhibit gluconeogenesis

Which of the following hormones is NOT secreted by the anterior pituitary gland:

a) Follicle-stimulating hormone b) Prolactin c) Oxytocin d) Luteinising hormone e) Growth hormone

Something wrong?

http://frcemsuccess.com/rev/sc­26/ 104/110 6/9/2017 Endocrine ­ FRCEM Success

The pituitary gland is primarily divided into two functional lobes, the anterior pituitary (adenohypophysis) and the posterior pituitary (neurohypophysis).

The following hormones are secreted from the anterior pituitary gland:

Follicle-stimulating hormone (FSH) Prolactin Thyroid-stimulating hormone (TSH) Adrenocorticotropic hormone (ACTH) Luteinising hormone (LH) Growth hormone (GH)

This can be remembered using the mnemonic: Fresh Pituitary Tastes Almost Like Guinness.

Antidiuretic hormone (ADH) and Oxytocin are secreted from the posterior pituitary gland.

The following pituitary hormones are matched with their main target tissue(s):

ACTH – Adrenal cortex GH – Liver, muscles, bones FSH – Gonads LH – Gonads Prolactin – Mammary glands TSH – Thyroid gland ADH – Kidney Oxytocin – Mammary glands, uterus

You are asked to assess a patient with hypernatraemia, they are dehydrated with a raised plasma osmolality and a serum Na+ of 160 mmol/L, which of the following is the most likely diagnosis:

a) Conn’s syndrome b) SIADH c) Near-drowning d) Addison’s disease e) Osmotic diuresis in uncontrolled diabetes mellitus

Something wrong?

Hypernatraemia is an increase in the serum sodium concentration above the reference interval of 133 – 146 mmol/L. Hypernatraemia may arise from either excess sodium or much more commonly from water decit (both are associated with a raised plasma osmolality).

Hypernatraemia can be thought of in relation to actual total body sodium. The most common group of patients are http://frcemsuccess.com/rev/sc­26/ 105/110 6/9/2017 Endocrine ­ FRCEM Success

those with hypernatraemia with decreased body sodium through loss of both water and sodium, but with a greater proportion of water loss which may result from:

renal losses e.g. osmotic diuresis in uncontrolled diabetes mellitus, loop diuretics, renal disease skin losses e.g. burns, excessive sweating in hot climate or exercise gastrointestinal losses e.g. vomiting, diarrhoea, stulae

Patients with hypernatraemia with normal total body sodium have a pure water decit which may result from:

inadequate water intake e.g. unconscious patient, dementia, disordered thirst perception in hypothalamic lesion excessive pure water loss e.g. diabetes insipidus (where plasma osmolality > urine osmolality)

Hypernatraemia with an actual increase in total body sodium is rare. Mild true hypernatraemia may be caused by primary hyperaldosteronism, but this is not typical of Conn’s syndrome, other causes include acute salt poisoning e.g. intravenous sodium bicarbonate, hypertonic saline, high sodium feeds in infants, near drowning in salt water.

When assessing patients with hypernatraemia:

If the hypernatraemia is mild (Na ≤ 150 mmol/L) and the patient has obvious signs of dehydration it is likely the ECF volume is reduced and that the patient has lost both sodium and water. Treatment should aim to replace the decit of uid by infusing isotonic saline, or if the decit is large, hypotonic saline. With more severe hypernatraemia (150 – 170 mmol/L), pure water loss is likely if the clinical signs of dehydration are mild in relation to the degree of hypernatraemia – this is because pure water loss is distributed evenly throughout the body compartments and the sodium content of the ECF is unchanged. Treatment should be aimed at replacing water either orally, or with 5% dextrose. With gross hypernatraemia ( > 180 mmol/L), an excess of sodium is likely, the patient may present with signs of uid overload. Treatment may be with diuretics, or rarely, by renal dialysis.

What is the primary aim of parathyroid hormone (PTH):

a) Increase plasma Ca2+ b) Decrease plasma Ca2+ 3- c) Increase plasma PO4 3- d) Decrease plasma PO4 e) Increase plasma Mg2+

Something wrong?

Parathyroid hormone (PTH) is synthesised by the chief cells in the parathyroid gland. PTH is released in response to falling plasma ionised calcium levels and increasing blood phosphate levels (indirectly by its binding to ionised calcium and thereby effective reduction of blood calcium levels).

PTH release is inhibited by normal blood calcium levels and hypomagnesaemia. http://frcemsuccess.com/rev/sc­26/ 106/110 6/9/2017 Endocrine ­ FRCEM Success

PTH acts to:

increase calcium and phosphate resorption from bone (via indirect upregulation of osteoclast activity) increase calcium reabsorption in the distal tubule of the nephron (by activating Ca2+ entry channels in the apical membrane and the Ca2+ ATPase pump on the basolateral membrane) increase phosphate excretion by inhibiting reabsorption in the proximal tubule of the nephron inhibit renal bicarbonate reabsorption stimulating a metabolism acidosis which favours dissociation of calcium from plasma proteins stimulate 1-alpha-hydroxylase in the kidneys to produce activated vitamin D indirectly increase calcium and phosphate absorption in the small intestine (via activated vitamin D)

Overall PTH acts to increase plasma calcium levels and decrease plasma phosphate levels.

Regarding the adrenal glands, which of the following statements is CORRECT:

a) The adrenal glands are located on the inferior pole of each kidney. b) The adrenal glands are intraperitoneal organs. c) The adrenal glands are enclosed in renal fascia. d) The adrenal medulla comprises about 60% of the gland. e) The adrenal gland is divided into two functionally distinct regions, the right and the left lobe.

Something wrong?

The adrenal glands are located on the superior pole of each kidney. The glands are retroperitoneal, covered in perinephric fat and enclosed in renal fascia. The adrenal gland is divided into two functionally distinct regions, the outer larger region (comprising about 90% of the gland) is called the adrenal cortex and the inner, much smaller region is called the medulla.

Catecholamines are secreted by which of the following:

a) Zona glomerulosa of the adrenal cortex b) Zona reticularis of the adrenal cortex c) Zona fasciculata of the adrenal cortex d) Adrenal medulla e) Anterior pituitary

Something wrong?

The adrenal cortex is controlled by the pituitary gland, responding to adrenocorticotrophic (ACTH) hormone.

It is functionally and anatomically divided into three zones of tissue which each secrete different steroid hormones: http://frcemsuccess.com/rev/sc­26/ 107/110 6/9/2017 Endocrine ­ FRCEM Success It is functionally and anatomically divided into three zones of tissue which each secrete different steroid hormones:

the outer zona glomerulosa which secretes mineralocorticoids (mainly aldosterone), the zona fasciculata which secretes glucocorticoids (mainly cortisol) the inner zona reticularis which secretes glucocorticoids and androgens (mainly dehydroepiandrosterone (DHEA))

The adrenal medulla produces catecholamines.

How does parathyroid hormone affect the kidney’s handling of phosphate:

a) Decreases phosphate reabsorption in the proximal tubule b) Increases phosphate secretion in the distal tubule c) Increases phosphate reabsorption in the distal tubule d) Increases phosphate reabsorption in the proximal tubule e) Decreases phosphate reabsorption in the distal tubule

Something wrong?

Parathyroid hormone (PTH) is synthesised by the chief cells in the parathyroid gland. PTH is released in response to falling plasma ionised calcium levels and increasing blood phosphate levels (indirectly by its binding to ionised calcium and thereby effective reduction of blood calcium levels).

PTH release is inhibited by normal blood calcium levels and hypomagnesaemia.

PTH acts to:

increase calcium and phosphate resorption from bone (via indirect upregulation of osteoclast activity) increase calcium reabsorption in the distal tubule of the nephron (by activating Ca2+ entry channels in the apical membrane and the Ca2+ ATPase pump on the basolateral membrane) increase phosphate excretion by inhibiting reabsorption in the proximal tubule of the nephron inhibit renal bicarbonate reabsorption stimulating a metabolism acidosis which favours dissociation of calcium from plasma proteins stimulate 1-alpha-hydroxylase in the kidneys to produce activated vitamin D indirectly increase calcium and phosphate absorption in the small intestine (via activated vitamin D)

Overall PTH acts to increase plasma calcium levels and decrease plasma phosphate levels.

What is the most likely diagnosis in a 34 year old man presenting with a hypochloraemic metabolic alkalosis, low serum renin and a mass in the adrenal gland:

a) Congenital adrenal hyperplasia b) Renal artery stenosis c) Cushing’s syndrome d) Conn’s syndrome http://frcemsuccess.com/rev/sc­26/ 108/110 6/9/2017 Endocrine ­ FRCEM Success d) Conn’s syndrome e) Addison’s disease

Something wrong?

Hyperaldosteronism causes sodium and water retention (with volume expansion and hypertension) with increased excretion of potassium and hydrogen ions.

Features of hyperaldosteronism include:

hypertension hypokalaemia metabolic alkalosis polyuria and polydipsia muscle weakness and spasm N.B. Hypernatraemia rarely occurs due to other regulating mechanisms.

Conn’s syndrome is the most common cause of primary hyperaldosteronism, in which patients have an adenoma of the zona glomerulosa of the adrenal cortex.

In primary hyperaldosteronism, renin levels will be low due to negative feedback. High renin levels suggest secondary hyperaldosteronism e.g. secondary to excessive diuretic therapy (most common cause), congestive heart failure, renal artery stenosis, nephritic syndrome or cirrhosis with ascites.

Regarding hypernatraemia, which of the following statements is CORRECT:

a) Hypernatraemia most commonly results from increased salt ingestion. b) Hypernatraemia is dened as a Na+ concentration of 140 mmol/L or above. c) Conn’s syndrome usually results in a severe refractory hypernatraemia. d) Hypernatraemia is usually associated with a decrease in plasma osmolality, e) Hypernatraemia may be caused by osmotic diuresis.

Something wrong?

Hypernatraemia is an increase in the serum sodium concentration above the reference interval of 133 – 146 mmol/L. Hypernatraemia may arise from either excess sodium or much more commonly from water decit (both are associated with a raised plasma osmolality).

Hypernatraemia can be thought of in relation to actual total body sodium. The most common group of patients are those with hypernatraemia with decreased body sodium through loss of both water and sodium, but with a greater proportion of water loss which may result from:

renal losses e.g. osmotic diuresis in uncontrolled diabetes mellitus, loop diuretics, renal disease http://frcemsuccess.com/rev/sc­26/ 109/110 6/9/2017 Endocrine ­ FRCEM Success skin losses e.g. burns, excessive sweating in hot climate or exercise gastrointestinal losses e.g. vomiting, diarrhoea, stulae

Patients with hypernatraemia with normal total body sodium have a pure water decit which may result from:

inadequate water intake e.g. unconscious patient, dementia, disordered thirst perception in hypothalamic lesion excessive pure water loss e.g. diabetes insipidus (where plasma osmolality > urine osmolality)

Hypernatraemia with an actual increase in total body sodium is rare. Mild true hypernatraemia may be caused by primary hyperaldosteronism, but this is not typical of Conn’s syndrome, other causes include acute salt poisoning e.g. intravenous sodium bicarbonate, hypertonic saline, high sodium feeds in infants, near drowning in salt water.

When assessing patients with hypernatraemia:

If the hypernatraemia is mild (Na ≤ 150 mmol/L) and the patient has obvious signs of dehydration it is likely the ECF volume is reduced and that the patient has lost both sodium and water. Treatment should aim to replace the decit of uid by infusing isotonic saline, or if the decit is large, hypotonic saline. With more severe hypernatraemia (150 – 170 mmol/L), pure water loss is likely if the clinical signs of dehydration are mild in relation to the degree of hypernatraemia – this is because pure water loss is distributed evenly throughout the body compartments and the sodium content of the ECF is unchanged. Treatment should be aimed at replacing water either orally, or with 5% dextrose. With gross hypernatraemia ( > 180 mmol/L), an excess of sodium is likely, the patient may present with signs of uid overload. Treatment may be with diuretics, or rarely, by renal dialysis.

FRCEM Success Resources

We are an online revision The Royal College of Advanced Life Support resource for FRCEM Primary Emergency Medicine Group and Intermediate exam Irish Association for Emergency Medicine preparation. Emergency Medicine Journal Advanced Trauma Life Lifeinthefastlane Terms & Conditions Support Instant Anatomy Get in Touch Resuscitation Council (UK) Patient.co.uk TeachMeAnatomy Trauma.org Radiopaedia

©2014 - 2017 FRCEM Success | Website designed & hosted by Cyberfrog Design

http://frcemsuccess.com/rev/sc­26/ 110/110